5 Gram Positive & Gram Negative Cocci (118) Flashcards

1
Q

Case
A previously healthy 24-year-old woman presented with high fever of sudden onset, severe myalgia, headache, vomiting, diarrhea, and diffuse rash resembling sunburn. She was on the 4th day of her menses and was using tampons. Her blood pressure was noted as 70/40. As part of laboratory investigations, blood, urine, and vaginal cultures were done. According to the microbiology report, heavy growth of Staphylococcus aureus was obtained from vaginal culture. Preliminary report of her blood culture showed no growth after 24 hours incubation. Microscopy of urine sediment showed 15 leukocytes per high power field. Urine culture did not yield any significant growth.

What staphylococcal toxin played a major role in causing this patient’s illness?

1 Staphylococcal alpha toxin
2 Panton Valentine leukocidin
3 Exfoliative toxin
4 Enterotoxin B
5 Toxic shock syndrome toxin

A

Toxic shock syndrome toxin 1

Toxic Shock Syndrome Toxin 1 (TSST1) plays a major role in causing the clinical manifestations of toxic shock syndrome in the patient. Exotoxins constitute essential components of the virulence mechanisms of Staphylococcus aureus. Nearly all strains secrete hemolysins, nucleases, proteases, lipases, hyaluronidase, and collagenase, which convert host tissue into nutrients required for bacterial growth. Some strains produce additional exoproteins that may be responsible for particular clinical manifestations. The clinical picture of this patient is suggestive of menstrual toxic shock syndrome (mTSS) associated with tampon use. The patient has vaginal colonization/infection with Staphylococcus aureus, and the multi-system involvement is due to intoxication. TSST1 is accepted as the cause of 100% of mTSS cases. It is a pyrogenic superantigen produced by about 20% Staphylococcus aureus strains and is the only one known to cause TSS from intra-vaginal sources, presumably due to its unique capacity to cross mucosal surfaces. TSST-1 interacts simultaneously with MHC class 2 molecules on the surface of antigen-presenting cells and T-cell receptors forming a tri-molecular complex. This induces profound T-cell proliferation, resulting in massive release of bioactive cytokines, causing TSS. In patients with mTSS, absent or insufficient titers of neutralizing antibodies to the toxin have been observed. The exact role of tampons in mTSS is not clear. Tampons are not the source of toxigenic Staphylococcus aureus. TSS unrelated to menses can occur and is associated with conditions like post-surgical and post-partum infections. In addition to TSST1, some of the Staphylococcal enterotoxins with superantigenic activity have been found to contribute to the pathogeneses of nonmenstrual TSS.
Alpha toxin of Staphylococcus aureus is a heterogenous protein, which can act on a broad spectrum of eukaryotic cell membranes. It is a potent hemolysin. It is not mainly responsible for toxic shock syndrome.
Panton Valentine leukocidin is a cytotoxin that causes leukocyte destruction and necrosis. About 3% of Staphylococcus aureus strains produce PV leukocidin, and they often produce necrotic lesions such as furunculosis and necrotizing pneumonia.
Exfoliative toxins are epidermolysins and split the epidermis between 2 cell layers, resulting in desquamation. There are 2 distinct proteins: epidermolytic toxin A, which is a chromosomal gene product that is heat stable, and epidermolytic toxin B, which is plasmid mediated and heat stable. These are superantigens and produce generalized desquamation of Staphylococcal scalded skin syndrome (SSSS).
Enterotoxin B is one of the Staphylococcal toxins that causes food poisoning. There are multiple enterotoxins produced by Staphylococcus aureus. Approximately 50% of strains can produce 1 or more of them. The toxin is heat stable and can resist the action of gut enzymes. Food poisoning results from ingestion of preformed toxin in food that has been contaminated with Staphylococcus aureus. It is a self-limiting condition. The emetic effect of the toxin depends on the activation of medullary emetic center through vagal and sympathetic stimulation. Enterotoxin B also possesses superantigenicity, which refers to the ability of the toxin to activate T lymphocytes without regard for the antigen specificity of these cells.

How well did you know this?
1
Not at all
2
3
4
5
Perfectly
2
Q

Case
A mother brings in her 5-year-old son due to papular and pustular lesions on his face. A serous honey-colored fluid exudes from the lesions. You suspect impetigo. A Gram stain reveals spherical gram-positive arrangements in irregular grape-like clusters.

What organism is most likely causing this patient’s condition?

1 Staphylococcus epidermidis
2 Staphylococcus aureus
3 Peptostreptococcus
4 Streptococcus pneumoniae
5 Haemophilus influenzae

A

Staphylococcus aureus

The history and lab findings suggest the diagnosis of impetigo, in which Staphylococcus aureus is likely the causative organism. The most common causes of impetigo are usually more than likely Staphylococcus aureus, but also may be beta-hemolytic streptococcus group A.
S. aureus causes inflammatory and toxin-mediated diseases. When a Gram stain has been completed, S. aureus will appear as spherical gram-positive cocci arranged in irregular grape-like clusters, as is described in this patient. Inflammatory diseases that can be caused by an active S. aureus infection include the following:
skin infections, including impetigo, furuncles, carbuncles, and cellulitis, surgical wound infections, eyelid infections, and postpartum breast infections
septicemia (sepsis), which can originate from any localized lesion, especially wound infection, or as a result of intravenous drug abuse
endocarditis on normal or prosthetic heart valves
osteomyelitis and arthritis, either hematogenous or traumatic
pneumonia in postoperative patients or following viral respiratory infection, especially influenza
abscesses (metastatic) in any organ
Streptococcus pneumoniae are gram-positive lancet-shaped cocci arranged in pairs (diplococci) or short chains. On blood agar they produce alpha-hemolysis. Virulence factors of pneumococci are polysaccharide capsules. Pneumococci cause pneumonia, bacteremia, meningitis, and infections of the upper respiratory tract such as otitis and sinusitis. Mortality rate is high in elderly, immunocompromised (especially splenectomized), or debilitated patients. They should be immunized with the polyvalent polysaccharide vaccine.
Peptostreptococci are anaerobic gram-positive cocci. It grows under anaerobic or microaerophilic conditions and produces variable hemolysis. Peptostreptococcus is a member of the normal flora of the gut and female genital tract, participating in mixed anaerobic infections of the abdomen, pelvis, and brain.
Staphylococcus epidermidis is part of normal human flora on the skin and mucous membranes, but it can cause infections of intravenous catheters and prosthetic implants. This organism is particularly infectious in the inpatient hospital setting in patients with compromised immune systems. Gram stain of this bacterium reveals gram-positive cocci arranged in grape-like clusters. Although the Gram stain characteristics are similar to the correct answer, the patient scenario is inappropriate.
Haemophilus influenzae is an incorrect choice in this scenario since observed Gram stain characteristics include gram-negative and rod-shaped.

How well did you know this?
1
Not at all
2
3
4
5
Perfectly
3
Q

Case
A 25-year-old woman residing in a rural area of Alaska delivered her first born at home. Within 48 hours, the newborn baby boy developed lethargy, poor feeding, and respiratory distress and was brought to the pediatric emergency department of the nearest hospital. He was admitted to the neonatal ICU with a clinical diagnosis of neonatal sepsis. Samples were collected for laboratory investigations including blood for culture. The infant was placed on empirical antibiotic therapy and other supportive measures. After overnight incubation at 37°C, Gram stain of blood culture showed Gram-positive cocci in chains. Subculture on 5% sheep blood agar medium grew grayish white colonies with a narrow zone of beta hemolysis. Further studies of the isolate showed that it was catalase and oxidase negative, hydrolized hippurate, and was bile-aesculin negative. It was resistant to bacitracin and gave a positive CAMP test (Christie Atkins-Munch-Petersen test). Presumptive identification of the organism could be made based on these tests. A serological test was used to confirm identification.

What is the most likely bacterium causing early onset neonatal infection?

1 Staphylococcus aureus
2 Streptococcus pyogenes
3 Streptococcus pneumoniae
4 Staphylococcus epidermidis
5 Streptococcus agalactiae

A

Streptococcus agalactiae

Streptococcus agalactiae is a Group B Streptococcus (GBS). GBS is a major cause of neonatal sepsis in the US and other western countries. The isolate from the blood culture has the typical characteristics of this bacterium. Camp factor produced by the organism is responsible for giving a positive CAMP test. CAMP test demonstrates synergistic hemolytic activity between GBS and staphylococcus aureus. An accentuated zone of hemolysis is seen when GBS is inoculated perpendicular to a streak of staphylococcus aureus on blood agar. S.aureus produces beta lysin, which partially lyses sheep RBCs and the camp factor enhances this hemolytic activity.
In developing countries with limited availability of streptococcal grouping sera, a positive CAMP test and positive hippurate hydrolysis are often used for presumptive identification of GBS and to differentiate it from streptococcus pyogenes (Group A Streptococcus), the most common hemolytic streptococcus associated with human infections.
Streptococcus pyogenes is usually bacitracin-sensitive and gives a negative CAMP test. Streptococcus pneumoniae (Pneumococcus) can be differentiated from GBS by its typical lanceolate diplococcal appearance and production of alpha hemolytic colonies on blood agar. Staphylococcus aureus and staphylococcus epidermidis appear as grape-like clusters of Gram-positive cocci and these are catalase positive.
The newborn infant referred to is having early onset neonatal sepsis caused by GBS (EOGBS disease). Neonates are considered to have EOGBS disease when clinical illness develops within 7 days after birth accompanied by GBS isolation from normally sterile sites. Late-onset infections occur in infants in their 2nd-12th week of life and are often obtained from the environment. In EOGBS disease, infection of the fetus results from ascending spread of GBS from the vagina of the mother, who is an asymptomatic carrier colonized with the organism. The gastrointestinal tract serves as the natural reservoir of GBS and is the likely source of colonization. Infection may cause septicemia, meningitis, or pneumonia in the newborn. Preterm infants are more susceptible to GBS infections. Conditions in the mother such as preterm labor, premature rupture of membranes, prolonged labor after the rupture of membranes, and GBS bacteriuria during pregnancy favor fetal infection. Infants become infected during passage through the birth canal.
GBS colonization of the vagina is often transient. It can be also intermittent or persistent. Collection of cultures between 35 and 37 weeks of gestation by swabbing both the lower vagina and rectum for detection of colonization by GBS in pregnant women and subsequent intrapartum antibiotic prophylaxis of colonized mothers is recommended by the Centers for Disease Control and Prevention for prevention of EOGBS infections. Intrapartum antibiotic prophylaxis, administered at least 4 hours before delivery, is found to reduce the incidence of EOGBS disease considerably. Intravenous penicillin G is the antibiotic of choice because of its narrow spectrum of activity. Ampicillin is considered as an alternative. In individuals at high risk of anaphylaxis to penicillin, erythromycin or clindamycin can be used. Cefazolin is used in women who have only mild allergic reaction to penicillin. Vancomycin is reserved for those colonized with GBS strains resistant to erythromycin and clindamycin and with a history of anaphylactic reaction to penicillin.
The conventional culture method to detect GBS colonization requires 36 to 72 hours. To overcome this slow turnaround time, antigen detection assays and nucleic acid detection assays have been explored. A PCR-based assay targeting cfb gene, which codes for the CAMP factor, is reportedly useful for detecting GBS DNA directly in vagino-rectal specimens. This test has the advantage of a turnaround time of 1-2 hours, but the disadvantage is its requirement for costly equipment. A PCR assay targeting scpB gene, which encodes C5a peptidase, is also reported to have given accurate results. A PCR-based assay for detection of GBS in neonatal blood samples is found to show good sensitivity and specificity.
Though increased use of intrapartum antibiotic prophylaxis has caused a significant decline in EOGBS disease, GBS still remains a leading cause of morbidity and mortality in the newborn. Intrapartum antibiotic prophylaxis also causes concern over the possibility of emergence of antibiotic resistance and neonatal infections by resistant Gram-negative bacteria. The need for a vaccine is felt and there is ongoing research in this field.
GBS strains associated with human infections possess a polysaccharide capsule which confers virulence to the organism. 9 capsular types have been identified, antibodies to which confer type-specific protection. Glycoconjugate vaccines against all 9 GBS serotypes have been shown to be immunogenic in animal models. Conjugate vaccines have been prepared using the capsular polysaccharides of GBS serotypes common in the US and have been shown to be highly immunogenic in healthy adults. It is reported that such vaccines are likely to be effective in preventing GBS disease in the at-risk populations, including neonates born to colonized mothers, peripartum women, diabetics, and the elderly with underlying illnesses. Surface proteins of GBS are found to elicit protective immunity, and therefore, are thought to be of relevance in vaccine production. Attempt to develop a universal protein-based candidate vaccine with a broader protective activity against various circulating serotypes of GBS is also in progress.
Staphylococcus aureus is a Gram-positive coccus that occurs in clusters. It produces large opaque beta hemolytic colonies with golden yellow pigment. It is catalase positive, coagulase positive, and can be easily differentiated from streptococci. Staphylococcal infections are among the most common bacterial infections, ranging from trivial superficial infections to life-threatening conditions such as septicemia and toxic shock syndrome. It is one of the common agents of nosocomial infections. Staphylococcus aureus may cause early-onset neonatal infections, though rarely.
Streptococcus pyogenes (Group A streptococcus) possesses a group-specific cell wall carbohydrate different from that of GBS. It can be distinguished from GBS by serological method in addition to the presumptive methods such as bacitracin sensitivity and CAMP test. The organism produces various types of enzymes and toxins, which contribute to its virulence and are responsible for a wide variety of suppurative infections of the skin and subcutaneous tissue, septicemia and pyemia, and the nonsuppurative sequelae, acute rheumatic fever, and acute glomerulonephritis. Early-onset neonatal infections associated with Streptococcus pyogenes are exteremely rare.
Streptococcus pneumoniae is a small Gram-positive lanceolate diplococcus, often capsulated, the capsule being one of the major virulence factors of the bacterium. It produces alpha hemolytic colonies on blood agar medium. Pneumococci colonize the human nasopharynx and can cause middle ear infections, sinusitis, conjunctivitis, and pneumonia. Meningitis is the most serious form of pneumococcal infections. The bacterium is also associated with acute exacerbations of chronic bronchitis. Isolation of streptococcus pneumoniae from early-onset neonatal infections is very rare.
Staphylococcus epidermidis is a coagulase-negative staphylococcus and is invariably present on the normal human skin. It is an opportunistic pathogen, causing infections when the host defenses are breached. It is implicated in causing nosocomial or late-onset neonatal sepsis. It is known to form biofilms on prosthetic devices and survive within them, inaccessible to antibiotics and host defenses. It can cause various infections, including cystitis, bacteremia, and endocarditis. Hospital strains are usually multidrug resistant.

How well did you know this?
1
Not at all
2
3
4
5
Perfectly
4
Q

Case
A 12-year-old boy was seen in the pediatric clinic for 2-day history of severe sore throat, pain and difficulty in swallowing, and high fever. On examination his temperature was 39.5°C and his tonsils were swollen and showed yellowish spots of exudates. The anterior cervical lymph glands were enlarged and tender. Throat swabs were collected and Rapid strep test was done. The test was positive. Culture of throat swab on sheep blood agar medium grew Gram-positive cocci in chains. The colonies were small with large zones of beta hemolysis. The bacterium isolated was sensitive to bacitracin. Presumptive identification was Streptococcus group A (Streptococcus pyogenes).

Which of the Strep pyogenes antigens listed below is detected by the rapid strep test?

1 Hyaluronic acid capsule
2 Streptolysin O
3 Streptolysin S
4 M protein
5 Cell wall carbohydrate antigen

A

Cell wall carbohydrate antigen

Rapid strep test is used for diagnosing sore throat caused by group A streptococcus. The test detects the cell wall carbohydrate antigen of the bacterium in the clinical sample.
Serological grouping of hemolytic streptococci is based on the nature of cell wall carbohydrate antigen (C substance). Rebecca Lancefield introduced the grouping in 1933 and the serogroups are also known as Lancefield’s groups. Groups A-U (without I and J) have been identified. Majority of human streptococcal infections are caused by group A streptococcus (GAS).

The group specific C carbohydrate antigen is an integral part of the cell wall. The cell wall is composed of an outer layer of protein and lipoteichoic acid, a middle layer of group specific carbohydrate, and an inner layer of peptidoglycan. The carbohydrate antigen has to be extracted by chemical methods to be detected using group specific antisera. Extraction of the antigen can be done by treating centrifuged culture with hot acids, by enzymatic lysis of streptococcal cells, or by autoclaving cell suspensions. The conventional methods of capillary or agar gel precipitation tests with group specific sera are used to identify the extracted carbohydrate antigen. Isolates of hemolytic streptococci belonging to other groups can also be identified by these methods. Co-agglutination and latex agglutination methods also have been used for grouping streptococci.

The serological specificity of the group-specific carbohydrate is determined by an amino sugar. For example, group-specific carbohydrate for group A streptococci is a polysaccharide chain consisting of repeat units of rhamnose capped by N-acetyl glucosamine molecules (rhamnose-N-acetyl glucosamine). The group specific carbohydrate for group B streptococcus is rhamnose-glucosamine polysaccharide and for group C, rhamnose-N-acetyl galactosamine.

Rapid antigen detection test done directly with the throat swab material also involves acid extraction of the group specific carbohydrate antigen of GAS from the clinical sample and identification of the antigen by immunological reaction. Different technologies are used like latex agglutination, enzyme immunoassay, optical immunoassay, and immunochromatographic assay. All are reported to have good specificity. False positive results are unusual and may be produced by the presence of Streptococcus milleri group of bacteria in the throat that express the group A carbohydrate antigen. Sensitivity of the test may vary with the method used. Though cultural isolation is the gold standard, positive rapid strep test is generally accepted for diagnosis of pharyngitis/tonsillitis caused by GAS. Early detection of GAS in throat swabs helps early administration of antibiotics and early prevention of suppurative complications and spread to others. When rapid test is negative, back up cultures help confirm diagnosis. Culture is required for performing susceptibility tests. Though no significant change in susceptibility to penicillin has been observed, erythromycin resistant strains and outbreaks due to such strains have been reported.

Recently, rapid antigen detection tests based on molecular methods (chemiluminiscent gene probes and polymerase chain reaction) have been developed, which require special equipment.

Streptococcal sore throat is the most common infection caused by hemolytic streptococcus. Virulent group A streptococcus adheres to the pharyngeal epithelium by means of lipoteichoic acid covering the surface pili and causes diffuse pharyngitis or localized tonsillitis. The bacterium is responsible for a variety of human infections ranging from milder diseases like pharyngitis and impetigo to severe life-threatening infections such as bacteremia, necrotizing fasciitis, and streptococcal toxic shock syndrome.

Various structural components of Streptococcus pyogenes show antigenic cross reaction with different tissues of the human body. Antigenic relationships have been observed between Group A carbohydrate and human cardiac valvular glycoproteins. The reactivity is related to N-acetyl glucosamine moiety present in both structures. Antigenic cross reactions with human tissues are considered to be of importance in the pathogenesis of acute rheumatic fever and acute glomerulonephritis, the non-suppurative sequelae of GAS infection. In these conditions the tissue damage produced is of immunological nature.

Hyaluronic acid capsule: Group A streptococci produces hyaluronic acid capsule better noticeable in young cltures. It is composed of equimolar concentrations ofN-acetyl glucosamine and glucuronic acid and is structurally identical to the hyaluronic acid of mammalian tissues. When present, the capsule inhibits phagocytosis. It is not antigenic in humans.
Streptolysin O: Streptolysins O and S are hemolysins and extra cellular products of hemolytic streptococci. Streptolysin O is oxygen labile. It is rapidly inactivated in presence of oxygen, hence the name. It resembles the oxygen-labile hemolysins of Clostridium perfringens and Cl.tetani and contributes to the virulence of the bacterium. Following Strep pyogenes infection, antibody is produced to streptolysin O. This antibody (anti-streptolysin O) inhibits hemolysis by streptolysin O. In rheumatic fever and acute glomerulonephritis, a retrospective diagnosis of streptococcal infection is helpful. This is done by demonstrating high levels of antibodies to streptococcal toxins. Quantitative estimation of anti streptolysin O (ASO titer) is a standard serological procedure for such retrospective diagnosis. An ASO titer in excess of 160-200 units is considered significant and suggestive of either recent or recurrent infections with streptococci.
Streptolysin S is the cytolytic factor, which causes the beta hemolytic zone surrounding the colonies on blood agar medium. It is elaborated in presence of serum, hence the name. It is oxygen stable. Streptolysin S is not immunogenic in humans and neutralizing antibodies are not evoked during infection. Sera from persons with past infection with streptococcus pyogenes do not neutralize streptolysin S. Non-specific inhibitors present in human sera may inhibit its activity. Streptolysin S is also considered as an important virulence determinant of group A streptococcus.
M protein: This is a major virulence factor of Strep pyogenes and is associated with resistance to phagocytosis and adherence to host cells. It is one of the protein antigens identified in the outer part of the cell wall. The other proteins are T and R. M protein is antigenic. Based on the M protein, GAS can be sero- typed. There are 2 major structural classes of M protein class I and class II. Class I M protein is thought to be a virulence determinant for rheumatic fever as the antigenic domains of this cross-react with cardiac muscle.
A few M serotypes (5, 14, 18, and 24) have been identified with outbreaks of acute rheumatic fever. A recent study of streptococcal pharyngitis in children conducted in the US has shown marked decrease in M-protein rheumatogenic types and increase in non-rheumatogenic types of S.pyogenes isolates. It is suggested that the near-disappearance of acute rheumatic fever in the US could be due to the replacement of rheumatogenic types of S.pyogenes by non-rheumatogenic types.
The association of M types with certain conditions is as follows:

  1. Streptococcal toxic shock syndrome-M protein types 1 and 3, which produce pyrogenic, exotoxins.
  2. Strains belonging to M types 12, 2, 4, and 49 are known to be nephritogenic.
  3. Skin infections by strep pyogenes M types 49, 57, and 59-61; strain characterization of GAS is mainly based on identification of M protein and is of importance in epidemiological and surveillance studies. To identify the M protein type, a genotyping system (emm typing) has been developed. It is based on the sequence of emm gene, which encodes for M protein. This method gives better strain identification and could be used also for identifying isolates non-typeable by M serotyping.
    Most common emm types in the US are reported to be 1, 28, 12, 3, and 11.
How well did you know this?
1
Not at all
2
3
4
5
Perfectly
5
Q

The Lancefield classification for Streptococci is based on

1 Clinical properties
2 Hemolytic properties
3 Serologic properties
4 Morphologic properties
5 Biochemical properties

A

Serologic properties

Lancefield grouping A through H, K, and V is based on serological properties of pathogenic strains possessing group specific antigens. These are detected by immunologic probes and are used for rapid identification of the streptococcal pathogens.
Hemolytic properties are also used to classify Streptococci as:
α-hemolytic - Incomplete hemolysis
β-hemolytic - Complete hemolysis
γ-hemolytic - No hemolysis

How well did you know this?
1
Not at all
2
3
4
5
Perfectly
6
Q

Streptococcus pyogenes causes pharyngitis and scarlet fever and is classified as

1 Group A
2 Group B
3 Group C
4 Group D
5 Group G

A

Group A

Lancefield grouping A through H, K, and V is based on serological properties of pathogenic strains possessing group specific antigens.
Streptococcus pyogenes is β-hemolytic Group A Streptococci .
Group B includes Streptococcus agalactiae.
Group C includes Streptococcus anginosus and Streptococcus equisimilis.
Group D includes Streptococcus bovis and Enterococcus spp.
Group F includes Streptococcus anginosus.

How well did you know this?
1
Not at all
2
3
4
5
Perfectly
7
Q

Virulence factors responsible for antiphagocytic and anticomplementary property of bacteria include
Answer Choices

1 Lipoteichoic Acid
2 M protein
3 F protein
4 Streptokinase
5 DNase

A

M protein

M protein protects the cell against phagocytosis in absence of antibodies. It also prevents interaction with complement.
Lipoteichoic acid and F protein mediates adherence to epithelial cells.
Streptokinase lyse blood clots.
DNase depolymerizes cell-free DNA in purulent material.

How well did you know this?
1
Not at all
2
3
4
5
Perfectly
8
Q

Streptolysin S. is

1 Oxygen stable, non-immunogenic
2 Oxygen labile, non-immunogenic
3 Oxygen stable, immunogenic
4 Oxygen labile, immunogenic

A

Oxygen stable, non-immunogenic

Streptolysin S. is oxygen stable, non-immunogenic cell-bound hemolysin capable of lysing erythrocytes, leukocytes, and platelets. It stimulates release of lysosomal contents after engulfment, with subsequent death of the phagocytic cell.
Streptolysin O. is inactivated reversibly by oxygen and irreversibly by cholesterol. Antibodies are readily formed against Streptolysin O.

How well did you know this?
1
Not at all
2
3
4
5
Perfectly
9
Q

The main cause of bacterial pharyngitis is

1 Streptococcus anginosus
2 Streptococcus bovis
3 Streptococcus pyogenes
4 Streptococcus agalactiae
5 Streptococcus pneumoniae

A

Streptococcus pyogenes

Group A Streptococcus or Streptococcus pyogenes is the major cause of bacterial pharyngitis with Group C and G occasionally involved.
Streptococcus anginosus is associated with conditions like endocarditis, dental caries and abscess formation in tissues.
Streptococcus bovis is associated with endocarditis, urinary tract infections and pyogenic infections.
Infections caused by Streptococcus agalactiae includes neonatal infections such as meningitis, pneumonia, bacteremia, and postpartum sepsis
Streptococcus pneumoniae causes pneumonia, sinusitis, otitis media, meningitis and bacteremia.

How well did you know this?
1
Not at all
2
3
4
5
Perfectly
10
Q

Scarlet fever is a complication of streptococcal infection and is preceded by
Answer Choices

1 Pharyngitis
2 Toxic Shock Syndrome
3 Rheumatic fever
4 Glomerulonephritis
5 Pneumonia

A

Pharyngitis

Scarlet fever is a complication of Streptococcal pharyngitis. It is seen when the organism is lysogenized by a temperate bacteriophage that stimulates production of pyogenic exotoxin.
A diffuse erythematous rash appears on the upper chest and then spreads to the extremities.
Rheumatic fever and acute glomerulonephritis are nonsuppurative complications of streptococcal infections.

How well did you know this?
1
Not at all
2
3
4
5
Perfectly
11
Q

The organism responsible for nonsuppurative sequelae, rheumatic fever, and acute glomerulonephritis is

1 Staphylococcus aureus
2 Corynebacterium diphtheriae
3 Streptococcus pyogenes
4 Micrococcus luteae
5 Listeria monocytogens

A

Streptococcus pyogenes

Rheumatic fever and acute glomerulonephritis is a non suppurative complication of Group A Streptococcal disease.
Rheumatic fever is characterized by inflammatory changes of the heart, joints, blood vessels, and subcutaneous tissues. This disease is associated with M18, M3, and M5 serotypes of group A Streptococcus.
Specific nephritogenic strains of group A streptococci are associated with acute glomerulonephritis, which is characterized by acute inflammation of the renal glomeruli with edema, hypertension, hematuria, and proteinuria.

How well did you know this?
1
Not at all
2
3
4
5
Perfectly
12
Q

Group A Streptococci are

1 Sensitive to optochin
2 Resistance to bacitracin
3 Sensitive to bacitracin
4 Positive for CAMP test
5 Able to hydrolyze hippurate and esculin

A

Sensitive to bacitracin

Identification of Group A Streptococci is based on following:
Blood agar plate β-hemolysis
Bacitracin Sensitive
Optochin Resistant
CAMP test Negative
Hydrolysis of hippurate Negative
Hydrolysis of esculin Negative

How well did you know this?
1
Not at all
2
3
4
5
Perfectly
13
Q

The organism known as a significant cause of septicemia, pneumonia and meningitis in newborns is

1 Group A Streptococcus
2 Group B Streptococcus
3 Group C Streptococcus
4 Group D Streptococcus
5 Group G Streptococcus

A

Group B Streptococcus

Group B Streptococcus or Streptococcus agalactiae colonizes the upper respiratory tract, lower gastrointestinal tract, and vagina. Infection, with subsequent development of disease in the neonates can occur in utero, at the time of birth leading to early onset disease. Early onset disease is characterized by septicemia, pneumonia and meningitis in newborns.
Late onset infection occurs between 1 week and 3 months after birth and is commonly manifested as meningitis.

How well did you know this?
1
Not at all
2
3
4
5
Perfectly
14
Q

α- hemolytic Streptococci

1 Produce clear zone of hemolysis
2 Lyse the RBC completely
3 Produce no zone of hemolysis
4 Produce green discoloration on blood agar
5 Produce blue pigment on blood agar

A

Produce green discoloration on blood agar

α-hemolytic streptococci produce green discoloration on blood agar due to incomplete hemolysis.
β-hemolytic streptococci cause complete lysis of RBC resulting in clear zone of hemolysis.
γ-hemolytic streptococci fail to lyse RBC, resulting in no zone of hemolysis.

How well did you know this?
1
Not at all
2
3
4
5
Perfectly
15
Q

Streptococci are classified based on hemolytic reaction on blood agar plates. Choose the correct statement

1 α-hemolytic Streptococci produce clear zone of hemolysis
2 α-hemolytic Streptococci produce yellow discoloration
3 β-hemolytic streptococci cause incomplete destruction of red blood cells
4 β-hemolytic Streptococci cause green discoloration
5 γ-hemolytic Streptococci produce no hemolysis

A

γ-hemolytic Streptococci produce no hemolysis

α-hemolytic Streptococci produce green discoloration due to incomplete hemolysis.
β-hemolytic Streptococci produce clear zone of hemolysis as a result of complete hemolysis.
γ-hemolytic Streptococci produce no zone of hemolysis.

How well did you know this?
1
Not at all
2
3
4
5
Perfectly
16
Q

The drug of choice for infection caused by Group A Streptococci (GAS) is

1 Vancomycin
2 Penicillin
3 Methicillin
4 Sulfonamides
5 Trimethoprim

A

Penicillin

Penicillin is the drug of choice for Group A Streptococci (GAS), since the organism is highly sensitive to this antibiotic.
For patients with history of rheumatic fever, prophylactic doses of penicillin are given to avoid any recurrent infections that may cause additional damage to the heart.
Erythromycin can be used for the patients with history of penicillin resistance.

How well did you know this?
1
Not at all
2
3
4
5
Perfectly
17
Q

The leading cause of Gram positive bacterial meningitis in neonates is

1 Streptococcus mutans
2 Streptococcus sanguis
3 Streptococcus pyogenes
4 Streptococcus agalactiae
5 Streptococcus pneumoniae

A

Streptococcus agalactiae

Streptococcus agalactiae is an important cause of neonatal disease. It occurs within several days postpartum where there is an association with sepsis, lung involvement and meningitis.
Streptococcus pyogenes is common cause of pharyngitis.
Streptococcus pneumoniae is the leading cause of lobar pneumonia.
Streptococcus mutans and Streptococcus sanguis are viridans streptococci commonly implicated in dental caries and subacute bacterial endocarditis.

How well did you know this?
1
Not at all
2
3
4
5
Perfectly
18
Q

The leading cause of subacute bacterial endocarditis is:

1 Streptococcus pyogenes.
2 H299Streptococcus agalactiae.
3 Viridans Streptococci.
4 Staphylococcus saprophyticus.
5 Staphylococcus epidermidis.

A

Viridans Streptococci.

Viridans Streptococci are normally non pathogenic and are normal flora of the upper respiratory tract but can infect the endocardium following rheumatic fever, causing subacute bacterial endocarditis. They are also commonly associated with dental caries and suppurative intra-abdominal infections.
Streptococcus pyogenes is common cause of pharyngitis.
Streptococcus agalactiae is an important cause of meningitis, pneumonia, and bacteremia in neonates.
Staphylococcus saprophyticus has a predilection for causing urinary tract infections in young, sexually active women.
Staphylococcus epidermidis is major cause of prosthetic valve endocarditis.

How well did you know this?
1
Not at all
2
3
4
5
Perfectly
19
Q

Gram positive, encapsulated, lancet shaped, α-hemolytic, optochin sensitive organism was isolated from sputum of a patient. The most probable organism is
Answer Choices

1 Streptococcus pyogenes
2 Streptococcus agalactiae
3 Enterococcus
4 Streptococcus pneumoniae
5 Viridans group streptococci

A

Streptococcus pneumoniae

Streptococcus pneumoniae is the leading cause of lobar pneumonia, it is optochin sensitive and bile soluble and resistant to bacitracin.
Streptococcus pyogenes and Streptococcus agalactiae are β-hemolytic and resistant to optochin however, Streptococcus pyogenes is sensitive to bacitracin.
Viridans group streptococci and Enterococcus are resistant to optochin as well as bacitracin.

How well did you know this?
1
Not at all
2
3
4
5
Perfectly
20
Q

Streptococcus pneumoniae is often confused with viridans streptococci. The characteristic that distinguish viridans streptococci from Streptococcus pneumoniae is that the viridans streptococci are

1 Sensitive to bacitracin
2 Resistance to optochin
3 Sensitive to optochin
4 Resistance to bacitracin
5 Bile soluble

A

Resistance to optochin

Streptococcus pneumoniae is optochin sensitive, bile soluble, and bacitracin resistant.
Viridans streptococci is resistance to optochin as well as bacitracin and is bile insoluble.

How well did you know this?
1
Not at all
2
3
4
5
Perfectly
21
Q

The most common cause of bacterial pneumonia as well as bacterial meningitis in the United States is

1 Streptococcus bovis
2 Streptococcus pyogenes
3 Neisseria meningitides
4 Klebsiella pneumoniae
5 Streptococcus pneumoniae

A

Streptococcus pneumoniae

In the United States Streptococcus pneumoniae is the most common cause of bacterial pneumonia (estimated 500,000 cases per year) as well as bacterial meningitis. It is also a common cause of otitis and sinusitis. The incidence of infections are highest in the children and elderly because these population have low levels of protective antibodies directed against pneumococcal capsular polysaccharides.

How well did you know this?
1
Not at all
2
3
4
5
Perfectly
22
Q

Streptococcus pneumoniae is

1 β-hemolytic, optochin sensitive, and catalase negative
2 β-hemolytic, optochin resistant, and catalase positive
3 α-hemolytic, optochin sensitive, and catalase negative
4 α-hemolytic, bacitracin sensitive, and catalase positive
5 γ-hemolytic, bacitracin resistant, and catalase negative

A

α-hemolytic, optochin sensitive, and catalase negative

Streptococci are catalase negative, gram positive cocci.
Streptococcus pneumoniae is encapsulated gram positive coccus that mostly appears in pairs and gives positive quellung reaction. The isolates give α-hemolytic colonies on blood agar plate, as a result of incomplete hemolysis. The colonies appear mucoid and become autolytic in older culture. The isolates are bile soluble and susceptible to optochin.

How well did you know this?
1
Not at all
2
3
4
5
Perfectly
23
Q

The virulence factor that contributes to autolytic property of Streptococcus pneumoniae is

1 Neuraminidase
2 Pneumolysin
3 Capsule
4 Amidase
5 Streptolysin O

A

Amidase

Amidase, the pneumoccal autolysin, hydrolyzes the peptidoglycan layer at the bond between N-acetylmuramic acid and alanine residue on the peptide cross-bridge.
The other virulence factors produced by Streptococcus pneumoniae are as follows:
Neuraminidase can cause the spread of the organism throughout infected tissue.
The capsular polysaccharide aids in inhibiting phagocytosis in the absence of specific antibodies.
Pneumolysin is dermotoxic and temperature and oxygen labile hemolysin.
Streptolysin O is oxygen labile hemolysin produced by Streptococcus pyogenes and is responsible for hemolysis on blood agar plates.

How well did you know this?
1
Not at all
2
3
4
5
Perfectly
24
Q

The drug of choice for pneumococcal pneumonia is

1 Vancomycin
2 Penicillin
3 Bacitracin
4 Ceftriaxone
5 Rifampin

A

Penicillin

Penicillin is the drug of choice for pneumococcal pneumonia since it is mostly sensitive to this antibiotic. Erythromycin, cephalosporins and chloramphenicol are alternative choices for penicillin allergic patients.

How well did you know this?
1
Not at all
2
3
4
5
Perfectly
25
Q

Enterococci are

1 Gram positive, catalase positive, and can grow in the presence of bile and sodium chloride
2 Gram positive, catalase negative, and cannot grow in the presence of bile and sodium chloride
3 Gram negative, catalase negative, and can grow in the presence of bile and sodiumchloride
4 Gram positive, catalase negative and can grow in the presence of bile and sodium chloride
5 Gram negative, catalase positive, and can grow in the presence of bile and sodium chloride

A

Gram positive, catalase negative and can grow in the presence of bile and sodium chloride

Enterococci are gram positive cocci found in pairs or short chains and are catalase negative. They can grow in the presence of high concentration of bile and sodium chloride and can survive in bowel and gall bladder.
Enterococcus faecalis is common isolate that is found in upper respiratory tract, small intestine, and large intestine.

How well did you know this?
1
Not at all
2
3
4
5
Perfectly
26
Q

Enterococcus (Streptococcus) faecalis possess the cell wall antigen of

1 Group A Streptococcus
2 Group B Streptococcus
3 Group C Streptococcus
4 Group D Streptococcus
5 Group F Streptococcus

A

Group D Streptococcus

Enterococcus faecalis was classified as Group D Streptococcus because they posses the group D cell wall antigen.
Group A includes Streptococcus pyogenes
Group B includes Streptococcus agalactiae
Group C includes Streptococcus anginosus and Streptococcus equisimilis.
Group D includes Streptococcus bovis.
Group F includes Streptococcus anginosus.

How well did you know this?
1
Not at all
2
3
4
5
Perfectly
27
Q

The most common species of enterococci responsible for human infections are

1 Enterococcus faecalis and Enterococcus faecium
2 Enterococcus sulfuncus and Enterococcus dispar
3 Enterococcus avium and Enterococcus raffinase
4 Enterococcus casseliflavus and Enterococcus dispar
5 Enterococcus durans and Enterococcus flavescenes

A

Enterococcus faecalis and Enterococcus faecium

Enterococcus faecalis and Enterococcus faecium accounts for majority of enterococcal infections in human These organisms are normal habitat of upper respiratory tract, small intestine and large intestine. Hospitalized patients with catheters or receiving broad spectrum antibiotics are at higher risk of contracting enterococcal infections. These organisms are associated with biliary and urinary tract infections, endocarditis, bacteremia and wound infections.

How well did you know this?
1
Not at all
2
3
4
5
Perfectly
28
Q

The staphylococcal structure that provides receptor for bacteriophage is

1 Peptidoglycan
2 Teichoic acid
3 Capsule
4 Protein A
5 Cytoplasmic membrane

A

Teichoic acid

Teichoic acid is complex, phosphate containing, species specific polysaccharides bound to both the peptidoglycan layer and cytoplasmic membrane. Attachment of staphylococci to mucosal surface is mediated by the cell wall teichoic acid through their specific binding to fibronectin. Teichoic acid regulates cationic concentration at cell wall membrane.
Teichoic acid is the receptor for bacteriophages. It also serves as attachment site for staphylothrombin, which catalyzes the conversion of fibrinogen to insoluble fibrin.

How well did you know this?
1
Not at all
2
3
4
5
Perfectly
29
Q

In Staphylococcus aureus, teichoic acid

1 Is the site of biosynthesis and respiratory enzymes
2 Stimulates production of endogenous pyogens
3 Inhibits opsonization and phagocytosis
4 Regulates cationic concentration at cell
wall
5 Binds Fc receptors

A

Regulates cationic concentration at cell wall

Teichoic acid is complex, phosphate containing species specific polysaccharides and is bound to both the peptidoglycan layer and cytoplasmic membrane. It aids in staphylococcal attachment to mucosal surface through by binding specifically to fibronectin. Teichoic acid also regulates cationic concentration at cell wall and provides receptor for bacteriophages.

How well did you know this?
1
Not at all
2
3
4
5
Perfectly
30
Q

Coagulase enzyme is produced by

1 Staphylococcus epidermidis
2 Staphylococcus saprophyticus
3 Streptococcus pyogenes
4 Staphylococcus aureus
5 Streptococcus pneumoniae

A

Staphylococcus aureus

Staphylococcus aureus produces enzyme coagulase that converts fibrinogen to fibrin. This organism possesses bound and free forms of coagulase. Coagulase bound to the staphylococcal cell wall can directly convert fibrinogen to insoluble fibrin and cause the organism to clump together. However, cell free coagulase causes the staphylococci to clump by reacting with a globulin plasma factor to form staphylothrombin, which catalyzes the conversion of fibrinogen to insoluble fibrin.

How well did you know this?
1
Not at all
2
3
4
5
Perfectly
31
Q

The enzyme produced by Staphylococci that is responsible for the conversion of toxic hydrogen peroxide to water and oxygen is

1 Lipase
2 Coagulase
3 Hyaluronidase
4 Collagenase
5 Catalase

A

Catalase

All Staphylococci produce catalase, an enzyme responsible for conversion of toxic hydrogen peroxide to water and oxygen that accumulates during bacterial metabolism or is released following phagocytosis.
Coagulase, lipase, hyaluronidase, and collagenase are exoenzyme produced by Staphylococci.
Coagulase is the exoenzyme produced by Staphylococcus aureus, and it clots the plasma.
Lipase hydrolyzes lipids and aids in the survival of staphylococci in the sebaceous glands.
Hyaluronidase enables pathogen to spread through connective tissue by breaking down hyaluronic acid, the “cement” that holds tissue cells together.
Collagenase breaks down collagen that is the supportive protein found in tendons.

How well did you know this?
1
Not at all
2
3
4
5
Perfectly
32
Q

An enzyme produced by Staphylococcus aureus that dissolves fibrin clots is

1 Hyaluronidase
2 Catalase
3 Staphylokinase
4 Lipase
5 Coagulase

A

Staphylokinase

Pathogenic bacteria release exoenzymes that increase their ability to invade body tissue. These include coagulase, kinases, lipase, hyaluronidase, and collagenase.
All Staphylococci produce catalase, an enzyme responsible for conversion of toxic hydrogen peroxide to water and oxygen that accumulates during bacterial metabolism or is released following phagocytosis.
Kinases dissolve fibrin clots, thus enabling the organism to invade and spread throughout the body. Staphylococcus aureus produces staphylokinase, which is fibrinolytic.
Coagulase is the exoenzyme produced by Staphylococcus aureus and clots the plasma.
Lipase hydrolyzes lipids and aids in the survival of staphylococci in the sebaceous glands.
Hyaluronidase enables pathogen to spread through connective tissue by breaking down hyaluronic acid, the “cement” that holds tissue cells together.
Collagenase breaks down collagen that is the supportive protein found in tendons.

How well did you know this?
1
Not at all
2
3
4
5
Perfectly
33
Q

The enzyme that facilitates the spread of Staphylococcus aureus in the connective tissue is

1 Lipase
2 Coagulase
3 Catalase
4 Lecithinase
5 Hyaluronidase

A

Hyaluronidase

Enzymes produced by staphylococci include coagulase, kinases, lipase, hyaluronidase, and collagenase.
Hyaluronidase hydrolyzes hyaluronic acids, the acidic mucopolysaccharides present in the acellular matrix connective tissue and facilitates the spread of Staphylococcus aureus in tissue.
Lipase hydrolyzes lipids and aids in the survival of staphylococci in the sebaceous glands.
Coagulase is the exoenzyme produced by Staphylococcus aureus and clots the plasma
Catalase converts hydrogen peroxide to water and oxygen.
Lecithinase is the enzyme produced by Clostridium perfringens that breaks down phospholipids.

How well did you know this?
1
Not at all
2
3
4
5
Perfectly
34
Q

Staphylococcal Scalded Skin Syndrome (SSSS) is seen in young children and is mediated by the action of

1 Alpha toxin
2 Exfoliative toxin
3 Leukocidin
4 Beta toxin
5 Enterotoxin A

A

Exfoliative toxin

Exfoliative toxin is implicated in staphylococcal scalded skin syndrome (SSSS), which is a spectrum of diseases characterized by exfoliative dermatitis. Exposure to this toxin leads to splitting of intracellular bridges (desmosomes) in the stratum granulosum layer of the outer epidermis.
Cognitive Level: Understand

How well did you know this?
1
Not at all
2
3
4
5
Perfectly
35
Q

Staphylococcal toxin mostly implicated in pseudomembrane enterocolitis is

1 Exfoliative toxin A
2 Exfoliative toxin B
3 Enterotoxin B
4 Enterotoxin C
5 Delta toxin

A

Enterotoxin B

Exfoliative toxin A and B are responsible for staphylococcal scalded skin syndrome (SSSS). Delta toxin is thermolabile toxin with cytolytic activity.
Enterotoxin B is associated with staphylococcal pseudomembrane enterocolitis.
Enterotoxin C and D are implicated in staphylococcal disease associated with contaminated milk products.

How well did you know this?
1
Not at all
2
3
4
5
Perfectly
36
Q

The enzyme responsible for penicillin resistance in staphylococci is

1 Nuclease
2 β-lactamase
3 Hyaluronidase
4 Coagulase
5 Catalase

A

β-lactamase

Resistance to penicillin is mainly developed as a result of β-lactamase (penicillinase) production which hydrolyzes β-lactam ring of penicillin. The genetic information encoding production of β-lactamase is carried on plasmids.
Hyaluronidase, nuclease, coagulase and catalase are staphylococcal enzymes implicated in virulence.

How well did you know this?
1
Not at all
2
3
4
5
Perfectly
37
Q

The antibiotic of choice for treatment of disease caused by Methicillin resistant Staphylococci aureus (MRSA) is:

1 Clindamycin
2 Nafcillin
3 Oxacillin
4 Penicillin
5 Vancomycin

A

Vancomycin

Number of penicillin resistant strains of Staphylococci aureus have increased significantly (approximately 90%). Methicillin (that is β-lactamase-resistant penicillin) was developed to treat penicillin resistant staphylococcal species.
However, a methicillin resistant strain of Staphylococci aureus (MRSA) and Staphylococci epidermidis (MRSE) has emerged.
Vancomycin is the antibiotic of choice when the organism is resistance to methicillin. This drug has adverse side effects and is used for patients suffering from bacteremia, endocarditis, and pneumonia.

How well did you know this?
1
Not at all
2
3
4
5
Perfectly
38
Q

The leading cause of urinary tract infection in young sexually active females is

1 Staphylococcus schleiferi
2 Staphylococcus intermedius
3 Staphylococcus delphini
4 Staphylococcus saprophyticus
5 Staphylococcus hyicus

A

Staphylococcus saprophyticus

Staphylococcus saprophyticus is coagulase negative staphylococci that is associated with urinary tract infection in young sexually active females. This species is found to adhere more effectively to the epithelial cells lining the urogenital tract than other coagulase negative staphylococci. Infected women usually have dysuria, pyuria and large numbers of organism in the urine. Staphylococcus schleiferi, Staphylococcus intermedius, Staphylococcus delphini and Staphylococcus hyicus are animal species and are rarely isolated from human samples.

How well did you know this?
1
Not at all
2
3
4
5
Perfectly
39
Q

Staphylococci causes localized pyogenic infections. Superficial infection affects mostly young children and manifests primarily on the face and limbs in the form of small macules that develop into pustules called

1 Furuncles
2 Carbuncles
3 Impetigo
4 Carbuncle
5 Erysipelas

A

Impetigo

Staphylococcus aureus causes cutaneous infections, which include impetigo, folliculitis, furuncles, and carbuncles.
Impetigo is usually caused by Group A Streptococcus alone or in combination with S. aureus. Folliculitis is a pyogenic infection localized to hair follicle.
Furuncles or boils are an extension of folliculitis.
Carbuncles result from coalescence of furuncles and extend to deeper subcutaneous tissue. Erysipelas is an acute superficial cellulitis of the skin with prominent lymphatic involvement and is preceded by either respiratory or skin infections, mostly with group A Streptococcus.

How well did you know this?
1
Not at all
2
3
4
5
Perfectly
40
Q

Furuncles are large, painful, raised nodules characterized by an underlying collection of dead and necrotic tissue. Furuncles are localized pyogenic staphylococcal infection and are an extension of

1 Impetigo
2 Folliculitis
3 Macule
4 Erysipelas
5 Carbuncle

A

Folliculitis

Staphylococcus aureus causes cutaneous infections, which include impetigo, folliculitis, furuncles, and carbuncles.
Furuncles or boils are an extension of Folliculitis.
Folliculitis is a pyogenic infection localized to hair follicle.
Carbuncle results from coalescence of furuncles and extend to deeper subcutaneous tissue. Chills and fever are associated with carbuncles and indicate systemic spread of staphylococci. Erysipelas is an acute superficial cellulitis of the skin with prominent lymphatic involvement and is preceded by either respiratory or skin infections, mostly with group A Streptococcus.

How well did you know this?
1
Not at all
2
3
4
5
Perfectly
41
Q

20% to 65% of all infection of prosthetic devices, catheters and shunts are caused by

1 Staphylococcus schleiferi
2 Coagulase negative staphylococci
3 Staphylococcus delphini
4 Staphylococcus intermedius
5 Staphylococcus hyicus

A

Coagulase negative staphylococci

The coagulase negative staphylococci have ability to produce a polysaccharide slime that can bond them to catheters and shunts and protect them from antibiotics and inflammatory cells.
Staphylococcus intermedius, Staphylococcus delphini and Staphylococcus hyicus are animal species and are rarely isolated from human samples.

How well did you know this?
1
Not at all
2
3
4
5
Perfectly
42
Q

The leading cause of septicemia following a surgical procedure is

1 Streptococci pneumoniae
2 Group B Streptococci
3 Escherichia coli
4 Staphylococcus aureus
5 Listeria monocytogenes

A

Staphylococcus aureus

More than half of Staphylococcus aureus bacteremia are acquired in the hospital following a surgical procedure or result from continued use of contaminated intravascular catheter. Staphylococci aureus bacteremias are associated with dissemination to other body sites, including the heart. Streptococci pneumoniae is a common cause of septicemia in children and in asplenic or sickle cell patients.
Group B Streptococci and Escherichia coli are the primary cause of septicemia in neonates. Listeria monocytogenes is a cause of septicemia in immunocompromised patients, alcoholics, and pregnant women.

How well did you know this?
1
Not at all
2
3
4
5
Perfectly
43
Q

The most common cause of acute bacterial endocarditis is

1 Staphylococcus saprophyticus
2 Staphylococcus aureus
3 Staphylococcus epidermidis
4 Streptococcus pneumoniae
5 Viridans streptococci

A

Staphylococcus aureus

Acute endocarditis caused by Staphylococcus aureus is a serious disease, with a mortality rate approaching 50%. Staphylococcus aureus endocarditis may be seen with nonspecific flu-like symptoms; however, the patient’s condition deteriorates rapidly with disruption of cardiac and peripheral evidence of septic embolization.
Staphylococcus saprophyticus is coagulase negative staphylococci that is associated with urinary tract infection in young, sexually active females.
Staphylococcus epidermidis is associated with prosthetic valve endocarditis.
Streptococcus pneumoniae can cause endocarditis in patients with normal or previously damaged heart valves.
Viridans streptococci are the common cause of subacute bacterial endocarditis.

How well did you know this?
1
Not at all
2
3
4
5
Perfectly
44
Q

Hematogenous osteomyelitis resulting from cutaneous infection in children is characterized by sudden onset of pain over the involved bone and high fever. The most common cause of hematogenous osteomyelitis is

1 Staphylococcus aureus
2 Staphylococcus saprophyticus
3 Staphylococcus intermedius
4 Eikenella corrodens
5 Pasteurella multocida

A

Staphylococcus aureus

Staphylococcus aureus osteomyelitis can be the result of hematogenous infection or secondary infection resulting from trauma or an overlying staphylococcal infection. Hematogenous spread in children usually involves the metaphyseal area of long bones.
Hematogenous osteomyelitis in adults usually occurs as vertebral osteomyelitis and is accompanied by intense back pain with fever.Staphylococcus saprophyticus is associated with urinary tract infection in young sexually active females.
Animal bite may cause Pasteurella multocida osteomyelitis; whereas, human bite may lead to infection by Eikenella corrodens.
Staphylococcus intermedius is rarely isolated from human samples.

How well did you know this?
1
Not at all
2
3
4
5
Perfectly
45
Q

The primary cause of septic arthritis in children over 2 years of age is

1 Neisseria gonorrhoeae
2 Escherichia coli
3 Streptococcus agalactiae
4 Staphylococcus aureus
5 Haemophilus influenzae

A

Staphylococcus aureus

Staphylococcal arthritis is characterized by a painful erythematous joint with purulence on aspiration. Staphylococcus aureus is also a primary cause of septic arthritis in adults receiving intra-articular injections or with mechanically abnormal joints. Infection is usually seen in large joints.
In sexually active persons, Neisseria gonorrhoeae is the most common cause of septic arthritis. Streptococcus agalactiae and Escherichia coli are common cause of neonatal sepsis.
Haemophilus influenzae is associated with infectious arthritis in the children less than 2 years of age.

How well did you know this?
1
Not at all
2
3
4
5
Perfectly
46
Q

The anaerobic gram positive cocci associated with brain and liver abscess is

1 Streptococcus pyogenes
2 Streptococcus intermedius
3 Streptococcus agalactiae
4 Staphylococcus aureus
5 Staphylococcus epidermidis

A

Streptococcus intermedius

Streptococcus intermedius and Streptococcus constellatus are anaerobic streptococci involved with abscesses and other purulent infection in soft tissue and internal organs.
Streptococcus intermedius produces hyaluronidase, DNase, and other extracellular enzymes, which may contribute to their pathogenicity.
Streptococcus pyogenes, Streptococcus agalactiae, Staphylococcus aureus, and Staphylococcus epidermidis are aerobic gram positive cocci.

How well did you know this?
1
Not at all
2
3
4
5
Perfectly
47
Q

Nutritionally variant streptococci give positive gram reaction and positive blood culture results but cannot be isolated when subcultured on regular media, as these organisms grow on medium supplemented with

1 Ascorbic acid
2 Bile esculin
3 6.5% Sodium chloride
4 Pyridoxal
5 Mannitol

A

Pyridoxal

Nutritionally variant streptococci (NVS) are a subgroup of viridans streptococci that are nutritionally deficient and can be isolated from patients with endocarditis and otitis media. These bacteria are called pyridoxal-dependent or vitamin B6-dependent streptococci. Most liquid and solid bacteriologic media do not contain vitamin B6, and therefore NVS cannot grow on these media. NVS may also be seen as satellites around colonies of staphylococci, Escherichia coli, Klebsiella spp, Enterobacter spp, and yeasts. In addition, the NVS are PYR positive, bile esculin negative, and do not grow on 6.5% NaCl. The viridans streptococci are PYR negative.

How well did you know this?
1
Not at all
2
3
4
5
Perfectly
48
Q

Gram-positive, nonmotile, spherical cell forming tetrads was isolated from the urine specimen of an immunocompromised patient. Other laboratory results were as follows:
Blood agar plate alpha hemolytic colonies
Hippurate Positive
PYR Positive
CAMP Negative
Leucine aminopeptidase Negative
Growth. in 6.5% NaCl Positive
The isolate was susceptible to vancomycin and bacitracin. The most probable organism isolated is

1 Aerococcus
2 Enterococcus
3 Pediococcus
4 Viridans streptococci
5 Nutritionally variant streptococci

A

Aerococcus

Aerococcus has been shown to cause endocarditis, urinary tract infections and other infections.
Aerococcus differs from streptococci in microscopic appearance. It does not form chains but forms tetrads. The colonies on blood agar are alpha hemolytic and resembles viridans streptococci. However, viridans streptococci are PYR negative, leucine aminopeptidase positive and do not grow in 6.5% NaCl. Pediococcus gives positive bile esculin and negative PYR reactions. Enterococcus can grow in 6.5% NaCl, but gives positive leucine aminopeptidase test and is resistant to bacitracin.

49
Q

What is the leading cause of purulent arthritis in a sexually active person?

1 Neisseria gonorrhoeae
2 Escherichia coli
3 Streptococcus agalactiae
4 Staphylococcus aureus
5 Staphylococcus saprophyticus

A

Neisseria gonorrhoeae

Blood-borne dissemination of Neisseria gonorrhoeae occurswith septicemia and infection of skin and joints in untreated asymptomatic infections. Clinical manifestation of disseminated disease includes fever, migratory arthralgias, suppurative arthritis in the wrists, knees, and ankles, and a pustular rash on an erythematous base over the extremities but sparing the head and trunk.
Staphylococcus aureus is the primary cause of septic arthritis in children more than 2 years of age
and in adults receiving intraarticular injections or with mechanically abnormal joints.
Streptococcus agalactiae and Escherichia coli are common cause of neonatal sepsis.
Staphylococcus saprophyticus is associated with urinary tract infection in young sexually active females.

50
Q

Biologic actions of Protein I associated with gonococci include

1 Antiphagocytic actions
2 Formation of surface pores
3 Attachment to human cells like epithelium of vagina
4 Hydrolysis of β-lactam ring in penicillin
5 Iron binding

A

Formation of surface pores

Protein I is a major surface antigen that forms surface pores. It also interfere with neutrophil degranulation.
Capsule of gonococci is associated with antiphagocytic actions.
Pili is responsible for attachment to human cells like epithelium of vagina, fallopian tube, and buccal cavity.
Iron binding protein binds iron required for gonococcal metabolism.
β-lactamase hydrolyzes β-lactam ring in penicillin.

51
Q

A gram stained smear of urethral exudate from male patients showing intracellular gram negative diplococci is diagnostic of infection with

1 Haemophilus ducreyi
2 Chlamydia trachomatis
3 Mycoplasma hominis
4 Neisseria gonorrhoeae
5 Ureaplasma urealyticum

A

Neisseria gonorrhoeae

The gram stain is very sensitive and specific test for detection of gonorrhea in men with purulent urethritis. Demonstration of gram-negative, intracellular, kidney bean shaped diplococci from a symptomatic man with discharge correlates at a rate of 95% with culture. However, in women the gram stain correlates in only 50-70% of cases with culture. Culture is performed for confirmation using selective media containing inhibiting agents like vancomycin, colistin and nystatin.
Haemophilus ducreyi is etiological agent of chancroid, a highly communicable sexually transmitted disease. Gram stain of genital lesions caused by H.ducreyi may show gram negative coccobacilli arranged in groups giving it “school of fish “ appearance.
Infection with organisms such as Chlamydia trachomatis or Ureaplasma urealyticum is suspected when gram stain shows more than five polymorphonuclear neutrophils per field but no bacteria.

52
Q

The antibiotic recommended by Centers for Disease Control (CDC) for treatment of gonorrhea is

1 Penicillin
2 Ceftriaxone
3 Erythromycin
4 Tetracycline
5 Aminoglycosides

A

Ceftriaxone

Penicillin is no longer the treatment of choice for following reasons:
1. The concentration of penicillin required to inhibit the growth of Neisseria gonorrhoeae has steadily increased.
2. There is an increase in β-lactamase-producing strain of Neisseria gonorrhoeae.
3. Chromosomal mediated resistant strain are resistant to penicillin as well as erythromycin, tetracycline and aminoglycosides.

53
Q

The genetic information for penicillinase producing strain of Neisseria gonorrhoeae is encoded on:

1 Composite transposons
2 Introns
3 Bacteriophage
4 Chromosome
5 Plasmids

A

Plasmids

In plasmid mediated penicillin resistance, the organism acquires a new plasmid with genes coding for β-lactamase production. Chromosomal mediated resistant strains do not produce β-lactamase and are resistant to penicillin as well as erythromycin, tetracycline, and aminoglycosides.
Chromosome mediated resistance is due to combination of mutation at several chromosomal loci.
Introns are DNA sequences not translated into gene products. Neisseria gonorrhoeae is a prokaryote and does not contain introns. Composite transposons carry the gene encoding kanamycin resistance.

54
Q

Neisseria gonorrhoeae is divided into five morphologically distinct colony types, T1 to T5 based on presence or absence of Pili. Select the statement that describes the colony types correctly

1 Type T1 and T2 possess pili
2 Type T3 and T4 possess pili
3 Type T1 and T2 are avirulent strains
4 Type T3 and T4 are virulent strains
5 Type T3 and T4 possess pili and are virulent strains

A

Type T1 and T2 possess pili

Neisseria gonorrhoeae Type T1 and T2 possess pili and are virulent forms of Neisseria gonorrhoeae. Pili helps in initial attachment of the organism to host tissue. Piliated strains facilitates genetic transformation and are resistant to phagocytosis. Type T3 , T4 and T5 are devoid of pili and are avirulent strains.

55
Q

Neisseria meningitidis is

1 Gram negative diplococci
2 Gram negative rods
3 Catalase positive and oxidase negative
4 Catalase negative and oxidase positive
5 Catalase negative and oxidase negative

A

Gram negative diplococci

With the exception of Neisseria elongata, all species of Neisseria are aerobic, gram negative diplococci, and catalase and cytochrome oxidase positive. Neisseria elongata is a catalase negative gram negative rod. Neisseria meningitidis is encapsulated and is differentiated from other Neisseria by acid production from glucose and maltose but not sucrose or lactose.

56
Q

Neisseria gonorrhoeae and Neisseria meningitidis are

1 Capnophiles
2 Obligate anaerobes
3 Motile
4 Facultative anaerobes
5 Oxidase negative

A

Capnophiles

Neisseria gonorrhoeae and Neisseria meningitidis are aerobic, non motile and oxidase positive gram negative diplococci. These organisms are capnophilic that is they grow better when atmosphere is enriched with carbon dioxide.

57
Q

Gram negative diplococci along with large number of polymorphonuclear neutrophils were seen in the cerebrospinal fluid of a 21yr old male having symptoms of purulent meningitis. The colonies on chocolate agar were catalase positive and oxidase positive. The isolate produced acid from glucose and maltose but not from sucrose and lactose. The causative organism is

1 Streptococcus pneumoniae
2 Streptococcus agalactiae
3 Neisseria meningitidis
4 Neisseria gonorrhoeae
5 Escherichia coli

A

Neisseria meningitidis

Streptococcus pneumoniae and Streptococcus agalactiae are catalase negative and gram positive cocci. Escherichia coli is oxidase negative gram negative rod. Neisseria gonorrhoeae and Neisseria meningitidis are gram negative diplococci and are catalase positive and oxidase positive. Neisseria meningitidis produces acid from glucose and maltose but not from sucrose and lactose. However, Neisseria gonorrhoeae produces acid from glucose only.

58
Q

The vaccine for Neisseria meningitidis is directed against

1 Group specific capsular polysaccharides
2 Lipooligosaccharide endotoxin
3 Pili
4 IgA protease

A

Group specific capsular polysaccharides

Group A, C, Y, and W135 capsular polysaccharides vaccines are developed to control infection with Neisseria meningitidis. Vaccination can be used for the following:
1. Controlling outbreak of the disease with a serogroup present in the vaccine
2. Individuals at higher risk of the infection.
3. Travelers to hyperendemic area.

59
Q

The most common non-pathogenic organism found to colonize the outer ear is

1 Catalase-negative Streptococcus
2 Catalase-positive Streptococcus
3 Coagulase-negative Staphylococcus
4 Coagulase-positive Staphylococcus
5 Oxidase-positive Neisseria

A

Coagulase-negative Staphylococcus

Coagulase-negative Staphylococcus is the most common organism found to colonize the outer ear. Other organisms colonizing the ear that are associated with disease in the ear include Streptococcus pneumoniae, Pseudomonas aeruginosa, and the Enterobacteriaceae.

60
Q

One of the most important criteria for identification of different Streptococci is the type of hemolysis they cause. Which one of the following species of Streptococci is only beta-hemolytic

1 S. pyogenes
2 S. faecalis
3 S. bovis
4 S. pneumoniae
5 Viridans group

A

S. pyogenes

The alpha hemolytic Streptococci such as S. pneumoniae and the viridans group form a green zone around their colonies on blood-agar plates due to incomplete lysis of red blood cells in blood- agar. Beta- hemolytic Streptococci form a clear zone around their colonies due to complete lysis of red blood cells. S. pyogenes is only beta- hemolytic. Gamma-hemolytic bacteria are non- hemolytic. S. faecalis can be alpha, beta or gamma hemolytic and S.bovis can be alpha or gamma- hemolytic.

61
Q

Which one of the following enzymes produced by Streptococci causes depolymerization of DNA in exudates or necrotic tissue

1 Streptodornase
2 Streptokinase
3 Hyaluronidase
4 Streptolysin O
5 Streptolysin S

A

Streptodornase

Streptodornase causes depolymerization of DNA in exudates or necrotic tissue. Streptokinase activates plasminogen to form plasmin which dissolves fibrin in clots, thrombi and emboli. Hyaluronidase hydrolyzes connective tissue and aids in the spread of Streptococci. Streptolysin O is oxygen labile and causes beta- hemolysis when colonies are grown on the surface of blood- agar plates. Streptolysin S is stable to oxygen and is responsible for and beta-hemolysis when colonies are grown on blood-agar plates.

62
Q

Which one of the following is an example of poststreptococcal disease?

1 Pharyngitis
2 Leukocytosis
3 Tender cervical nodes
4 Rheumatic fever

A

Rheumatic fever

Rheumatic fever is an example of a poststreptococcal infection that occurs 1-4 weeks after infection by S. pyogenes. It is characterized by fever, migratory polyarthritis and carditis due to immunological cross reaction between streptococcal antigens and antigens of joint and heart tissue. Pharyngitis, leukocytosis and tender cervical nodes are initial reactions to S. pyogenes infection.

63
Q

Which one of the following is an example of an anaerobic or microaerophilic streptococcus that is part of the normal flora of the gut and female genital tract and participates in mixed anaerobic infections?

1 Peptostreptococcus
2 S. mitis
3 S. mutans
4 S. pneumoniae
5 S. pyogenes

A

Peptostreptococcus

Peptostreptococcus is an example of an anaerobic or microaerophilic streptococcus that is part of the normal flora of the gut and female genital tract and participates in mixed anaerobic infections. S. mitis, S. mutans, S. pneumoniae, and S. pyogenes are aerobic streptococci and are not found in the gut or the female genital tract.

64
Q

Which one of the following sugars is fermented by gonococci?

1 Fructose
2 Glucose
3 Maltose
4 Acetate
5 Formate

A

Glucose

Gonococci ferment glucose and not fructose and maltose. Acetate and formate are not sugars.

65
Q

Thayer-Martin medium is used to culture gonococci and it contains

1 Chocolate agar
2 Antibiotics
3 Antibiotics and chocolate agar
4 Fungizone
5 Fungizone and chocolate agar

A

Antibiotics and chocolate agar

Thayer-Martin medium contains chocolate agar and the antibiotics vancomycin, colistin, trimethoprim and nystatin to inhibit growth of the normal flora of the genital tract. It does not contain fungizone.

66
Q

Mannitol salt agar (MSA) can be used in the laboratory when a specific potential pathogen is being sought from a specimen contaminated with other organisms. What organism is most likely to be isolated on MSA?

1 Escherichia coli

2 Pseudomonas aeruginosa

3 Enterococcus faecalis

4 Staphylococcus aureus

5 Streptococcus pyogenes

A

Staphylococcus aureus

Mannitol salt agar is designed to allow the halophilic (salt-loving) staphylococci to grow. This medium contains, in most formulations, 6.5% NaCl which is highly inhibitory to most organisms but will allow the staphylococci to grow. Most staphylococci are not inhibited in the presence of this concentration of salt. In fact, since the staphylococci are normal inhabitants of most skin surfaces, they must be adapted to the salt concentrations found in sweat. Mannitol is a carbohydrate incorporated into the MSA to differentiate S. aureus (positive) from other staphylococci (negative).
While it is possible for Enterococcus faecalis to grow on the mannitol salt agar, as it is relatively haloduric, these organisms are catalase negative.

67
Q

An organism growing on a blood agar plate is a gram-positive coccus and is catalase positive, but it is coagulase negative. Upon further testing to determine the genus of the organism, it is found to be resistant to a bacitracin disk (0.04 mg) and susceptible to a furazolidone disk (100 mg). The most likely organism growing on the plate is a

1 Micrococcus
2 Staphylococcus
3 Aerococcus
4 Enterococcus
5 Stomatococcus

A

Staphylococcus

Staphylococci and micrococci are gram-positive cocci that are catalase positive. Because the micrococci are coagulase negative, they may be incorrectly reported as coagulase-negative staphylococci. To distinguish between these two genera, two relatively easy tests to perform include the use of a bacitracin disk and a furazolidone disk. Staphylococci are resistant to bacitracin and susceptible to furazolidone. Micrococci are susceptible to bacitracin and resistant to furazolidone. Aerococci and enterococci are catalase negative. Stomatococci grow with an unusually “sticky” colony, quite unlike the staphylococci, and they are susceptible to bacitracin. Table 1 gives the reactions that would separate these organisms.
Table 1: Reaction of Gram Positive Cocci to Selected Tests
Organism Catalase Bacitracin Resistant Furazolidone Resistant
Staphylococcus + + -
Micrococcus + - +
Aerococcus - - -
Enterococcus - + -
Stomatococcus + or - - + or -

68
Q

The most widely used and generally accepted criterion for the identification of Staphylococcus aureus in the laboratory is

1 The ability to clot plasma
2 The ability to utilize mannitol
3 The ability to produce beta hemolysis
4 The ability to break down hydrogen peroxide
5 The ability to be lysed by specific phages

A

The ability to clot plasma

Most strains of S. aureus utilize mannitol, but there may be some negative strains. Some strains, but not all, produce beta hemolysis on 5% sheep blood agar. All strain are catalase positive (a breakdown of hydrogen peroxide), and some strains can be lysed by specific phages. However, the accepted defining characteristic equated to the definition of S. Aureus is that it is coagulase positive (clots plasma).

69
Q

Staphylococcus aureus, S. intermedius, S. hyicus, and S. schleiferi are related by the fact that they are all:

1 Beta hemolytic on 5% sheep blood agar
2 Common isolates from the normal nares
3 Coagulase positive
4 Oxidase positive by a modified oxidase test
5 Common isolates from apocrine gland areas

A

Coagulase positive

All 4 species (Staphylococcus aureus, S. intermedius, S. hyicus, and S. schleiferi ) are, in fact, coagulase positive. Although S. aureus is the common human pathogen in this group, the other 3 may rarely be isolated from human infection. S. hyicus is a common resident of ungulates and their dairy products. S. intermedius and S. schleiferi are common on carnivores; although, human isolates have been associated with some serious conditions. Only S. aureus commonly produces beta hemolysis, and this species is also the common isolate from the human nares. All human strains of staphylococci are negative in the modified oxidase test. S. hominis and S. hemolyticus are most common in areas of apocrine glands.

70
Q

In the laboratory, you observe that a technologist has taken an isolated colony from a blood agar plate, and made a heavy suspension of this organism in a drop of water on a glass slide. To this suspension, she has added one drop of rabbit plasma with an inoculating loop, and stirred the mixture. The mixture seemed to make a smooth, white suspension of cells but with little other reaction. She records a negative reaction on the laboratory work card. What is the gram reaction of the organisms that this technologist was testing?

1 Gram-positive cocci
2 Gram-negative cocci
3 Gram-positive rods
4 Gram-negative rods
5 Gram-negative, pleomorphic rods

A

Gram-positive cocci

This is a description of the slide coagulase test, sometimes called the clumping factor test, for differentiating S. aureus (coagulase positive) from other staphylococci considered coagulase-negative staphylococci. All staphylococci are gram-positive cocci.

71
Q

A patient presents with stable vital signs but with symptoms suggesting a deep-seated tissue infection, or perhaps osteomyelitis, for which blood cultures have shown no growth. To confirm your suspicion of a deep staphylococcal infection, one helpful test to order might be

1 A test for toxic shock syndrome toxin TSS-1
2 A test for teichoic acid antibodies
3 An EIA test for staphylococcal antigens in blood
4 A DNA probe for the Mec gene of S. aureus
5 A culture of a surgically obtained bone fragment directly from the infected site

A

A test for teichoic acid antibodies

Teichoic acids are major cell wall components of staphylococci, and some other gram-positive organisms, and they vary in structure depending on the organism. Antibodies to teichoic acids are usually produced in deep-seated staphylococcal infections such as osteomyelitis. Detection of antibodies to teichoic acids may be useful when blood cultures are nonproductive or when tissue specimens are not available for cultures. It is not a commonly performed test and must be done in reference laboratories. In this case, there was no described indication for toxic shock syndrome so a test for TSS-1 toxin might not be considered early. Currently there are no direct antigen tests for staphylococci in blood. Using a DNA probe for the gene for methicillin resistance requires an organism and one had not been cultured in this case. Although bone could be cultured (along with a blood culture) it is a highly invasive procedure that could possibly be avoided if the teichoic acid antibodies could be detected to confirm the suspicion of a staphylococcal etiology.

72
Q

In your hospital, there has been a rapid increase in the number of post-surgical infections due to S. aureus . Working with the infection control nurse, it is determined that nine of the twelve infected patients had been associated with surgical suite B, and that all nine of these patients had been in contact with one particular ICU nurse. If the laboratory has saved all of these isolates from the patients, and one isolate from the nose of the nurse, the best way to determine the relationship of these isolates to the outbreak is to request

1 A review of the antimicrobic susceptibility profiles (antibiogram) of the isolates
2 An evaluation of the biochemical patterns (biotype) of the isolates
3 A plasmid analysis (plasmid profile) of the isolates
4 Phage typing be performed on the isolates
5 Pulsed-field gel electrophoresis (PFGE) be performed on the isolates

A

Pulsed-field gel electrophoresis (PFGE) be performed on the isolates

The Centers for Disease Control and Prevention no longer recommends, or offers phage typing for staphylococcal isolates, since almost 20% of strains could not be phage typed. The method of choice for epidemiologic typing of staphylococci is PFGE, which is a chromosomal “fingerprint” that provides a pattern on all isolates. Antibiograms are helpful as a screening method but strains with the same antibiogram may be genetically unrelated by PFGE profiles. Biotypes are unreliable for epidemiology studies and plasmid profiles are less useful than PFGE because plasmids can be gained or lost so easily.

73
Q

A 7-month-old child presents to the emergency room exhibiting toxic epidermal necrolysis and large, flaccid bullae that are forming and rupturing under the epidermis, causing desquamation in the diaper area. This spectrum of disease describes a condition usually caused by which one of the following?

1 Toxin producing Staphylococcus aureus.
2 Invasive Escherichia coli.
3 Prophage infected Corynebacterium diphtheriae.
4 Flesh-eating Streptococcus pyogenes.
5 Dermal infection with Clostridium perfringens.

A

Toxin producing Staphylococcus aureus.

Scalded skin syndrome consists of three separate entities: toxic epidermal necrolysis, scarlatiniform erythema, and bullous impetigo, which appear to be a spectrum of disease. All three conditions are attributed to the action of the exfoliating toxin of S. aureus . Interestingly, S. aureus is often not recovered from the bullae, apparently because the toxin is produced at another site of infection. In bullous impetigo, also a disease of children, the bullae are localized and are filled with staphylococci and polymorphonuclear leukocytes. A thin crust forms over the sites where the bullae have ruptured.S. pyogenes may rarely cause a necrotizing fasciitis but not the typical scalded skin syndrome. C. diphtheriae may also cause dermal lesions but they are often found in patients such as the homeless, who often live in unsanitary condition. E. coli and C. Perfringens are not usually involved in skin lesions.

74
Q

Coagulase negative staphylococci, when isolated in the presence of other normal skin flora, are often considered nonpathogenic and may often be ignored as potential etiologic agents. However, in the presence of a foreign object, such as a central line catheter, these organisms tend to be more significant in some cases. Which characteristic of coagulase negative staphylococci tends to be related to pathogenesis in this circumstance?

1 The presence of the coagulase gene
2 Toxin production by plasmid-containing strains
3 Invasiveness due to extracellular enzymes
4 Production of extracellular slime
5 Teichoic acid activation of the inflammatory response

A

Production of extracellular slime

Slime production by the coagulase negative staphylococci (CNS), in the presence of indwelling medical devices, allows them to adhere to the devices. This glycocalyx is a matrix that not only protects the organisms, it allows them to remain in one area for a long period of time that can allow continuous seeding of bacteria into the bloodstream along with localized production of toxic products. The slime appears to inhibit neutrophil chemotaxis and phagocytosis and to inhibit the antimicrobial action of the vancomycin and teicoplanin. The source of most of these organisms is endogenous from the skin or mucosal surfaces of the patient or from the hands of medical personnel. The true pathogenic mechanisms for the CNS remain unclear.
Since CNS are, by definition, coagulase negative, a test for the coagulase gene would be meaningless and little is known about plasmid-directed toxin production by CNS strains. Generally, the CNS are not invasive. In fact, extracellular enzymes are more likely associated with invasiveness of gram-positive organisms such as the Clostridium sp. Teichoic acids have not been shown to be associated with infections from indwelling medical devices.

75
Q

The most frequently isolated coagulase negative staphylococcus (CNS) from bloodstream infections caused by CNS is

1 S. saprophyticus
2 S. haemolyticus
3 S. hominis
4 S. warneri
5 S. epidermidis

A

S. epidermidis

S. epidermidis is the most frequently isolated coagulase negative staphylococcus from bloodstream infection. It is estimated that these organisms account for 74% to 92% of hospital-acquired CNS bacteremia. S. haemolyticus, S. warneri, and S. hominis are also isolated from true infections but much less frequently. S. saprophyticus is usually considered a urinary tract isolate.

76
Q

Members of the genus Staphylococcus may be the etiologic agent of urinary tract infection, especially in young women. Which species is most likely to be considered a urinary tract pathogen in this population?

1 S. aureus
2 S. epidermidis
3 S. saprophyticus
4 S. hominis
5 S. warneri

A

S. saprophyticus

After E. coli, S. saprophyticus is the most frequent cause of urinary tract infections in young women. It can also cause urinary tract infections in men, usually over the age of 50.

77
Q

Which of the following is true for both scalded skin syndrome and scarlet fever?

1 Both diseases are caused by the same bacteria
2 Both diseases are toxin mediated
3 Toxin causes the skin to desquamate
4 A vaccine is available for each disease
5 Both organisms rely on a temperate phage for toxin production

A

Both diseases are toxin mediated

Both scalded skin syndrome (SSS) and scarlet fever are caused by toxins but from two different bacteria. SSS is due to toxins from S. aureus and scarlet fever from the toxins of Streptococcus pyogenes . Desquamation is associated with SSS but not usually with scarlet fever where the toxin is phage-mediated. Vaccines are not routinely available for either disease. Scarlet fever is usually one of the sequelae associated with recent untreated pharyngeal infection caused by S. pyogenes.

78
Q

The initial classification of beta-hemolytic streptococci into “groups” is probably used more commonly than the species name of the organisms. This classification of beta-hemolytic strains is based on

1 Immunologic characterization of carbohydrate antigens in the cell wall
2 Toxic characterization of strains based
on direct antigen detection
3 Immunologic characterization of lipoproteins in the cell membrane
4 Biochemical characterization derived from commercial identification systems
5 Catalase and coagulase characterization from initial isolation

A

Immunologic characterization of carbohydrate antigens in the cell wall

In the 1930’s, Rebecca Lancefield developed the classification scheme still in use today. Her approach was based upon immunologic characterization of carbohydrate and protein antigens present in the cell walls of streptococci. Her scheme originally was limited to beta-hemolytic isolates and were designated A-G. Later, alpha- and non-hemolytic streptococci were appended by other workers to the the Lancefield system and were assigned the designations H-O.
Neither toxin detection nor the immunologic characterization of lipoproteins play a role in the identification of streptococci. Biochemical methods are available, but many laboratories base their reports on the Lancefield grouping and not on biochemical identification. The catalase and coagulase test are used to characterize the staphylococci, although the streptococci are catalase negative. The coagulase test is not performed on streptococci.

79
Q

Leuconostoc spp. and Pediococcus spp. are members of the Streptococcaceae family that are occasionally isolated from humans but are often misidentified or considered normal flora because they are alpha-hemolytic. In some instances, however, it may be important to distinguish members of the genus Streptococcus from these other genera. The characteristic that will most likely separate these groups is

1 Leuconostoc and Pediococcus are actually short rods whereas the streptococci are true cocci
2 Streptococci are all positive in the PYR test while Leuconostoc and Pediococcus are both negative
3 Leuconostoc and Pediococcus are resistant to vancomycin while all streptococci are susceptible
4 Streptococcus spp are catalase negative while Leuconostoc and Pediococcus are catalase positive
5 Streptococci are bacitracin susceptible while the other genera are resistant

A

Leuconostoc and Pediococcus are resistant to vancomycin while all streptococci are susceptible

Leuconostoc spp and Pediococcus spp are inherently resistant to vancomycin while the streptococci are susceptible. These genera are gram-positive cocci resembling viridans streptococci in colonial morphology. All three genera are negative in the PYR test except for the Group A, beta-hemolytic streptococci which are PYR positive. In addition, all three genera are catalase negative because they all belong to the family Streptococcaceae. Bacitracin is used only on beta-hemolytic strains and would not be a factor in distinguishing these genera.

80
Q

Enzyme immunoassay kits used to perform direct antigen detection on patients with pharyngitis usually require an extraction step for what specific cellular component?

1 Cell wall protein
2 Cell membrane lipopolysaccharide
3 Cell wall carbohydrate
4 Specific teichoic acid moieties within the cell wall and cell membrane
5 A specific cell membrane-bound metabolic enzyme

A

Cell wall carbohydrate

EIA direct antigen tests for streptococci detect the cell wall carbohydrate designating Group A, B, or C strains, depending on the type of commercial test used. Because these tests are very specific, negative EIA tests for Group A, taken from symptomatic patients, should be cultured and examined for the beta-hemolytic colonies typical of other streptococci that can cause pharyngitis. These include Group C and G, which would be negative in the Group A direct antigen tests.

81
Q

By using a commercial medium containing 5% sheep blood + trimethoprim-sulfamethoxazole (SXT), the laboratory is able to separate which organism from other normal flora at the site?

1 Staphylococcus aureus
2 Streptococcus pneumoniae
3 Pseudomonas aeruginosa
4 Streptococcus pyogenes
5 Enterococcus faecalis

A

Streptococcus pyogenes

SXT-containing blood agar is available commercially and is frequently used to facilitate the isolation of Group A streptococci from throats. The SXT tends to inhibit most other normal throat microorganisms. Although some commensal flora may grow in the presence of SXT, the S. pyogenes beta-hemolytic colonies can easily be detected.

82
Q

Upon examination of a blood culture bottle that has been logged as positive by the automated blood culture instrument, there appears to be no growth on visual inspection, no growth on subculture at the appropriate temperature, but on gram-stain one can see gram positive cocci. The next step in trying to determine the identity of this potentially pathogenic organism is to

1 Centrifuge the broth and do a direct antigen test for Group A streptococcus on the sediment
2 Inoculate the broth from the blood culture bottle onto mannitol salt agar and incubate in carbon dioxide
3 Plate the broth on chocolate agar and place an Optochin disk over the streak
4 Do a slide coagulase test on the centrifuged sediment from the blood culture bottle
5 Plate the broth on blood agar and place a staphylococcus streak down the middle of the inoculum

A

Plate the broth on blood agar and place a staphylococcus streak down the middle of the inoculum

The most likely cause for this observation in blood culture bottles, would be the presence in the broth of nutritionally variant streptococci. When present, these organisms may or may not be seen on visual examination, the instrument will indicate that the bottle is truly positive, and the gram-stain will confirm the presence of a gram-positive coccus. However, because these organisms require pyridoxal (vitamin B6), they will not grow on routine media. The pyridoxal must be supplied either by the addition of the compound or by cocultivation with a staphylococcus that produces it. In the latter case, the streptococci will be seen “satelliting” around the growth of the staphylococci.

83
Q

A test for the production of pyrrolidonyl arylamidase may be used to specifically identify which organism?

1 Staphylococcus aureus
2 Stenotrophomonas maltophilia
3 Serratia marcescens
4 Streptococcus pyogenes
5 Salmonella enteritidis

A

Streptococcus pyogenes

The PYR test can be used instead of the bacitracin susceptibility test to identify Group A streptococci, such as Streptococcus pyogenes . It is just as sensitive as the bacitracin test but is considerably more specific. In addition, it is a rapid 10-minute procedure. The PYR test is also used to recognize members of the genus Enterococcus.

84
Q

Streptococcus agalactiae is a potential life-threatening pathogen in newborns. It can be cultured from vaginal specimens taken prior to delivery, as well as from cultures taken from the newborn. One test that can identify this isolate with a high degree of accuracy is the

1 CAMP test
2 PYR test
3 Bacitracin test
4 Indole test
5 Vancomycin test

A

CAMP test

The CAMP test detects production of a diffusible, heat-stable extracellular protein that enhances the hemolysis of sheep erythrocytes by the beta hemolysin of Staphylococcus aureus. A positive result is recognized by the appearance of a zone of synergistic beta hemolysis between the growth of S. aureus and S. agalactiae (the Group B streptococcus Group B streptococci are the only beta-hemolytic streptococci that yield a positive CAMP test).
The PYR test is for Group A streptococci and the enterococci. Bacitracin is used to detect Group A streptococci. The indole test is typically used for gram-negative rods and the vancomycin susceptibility test; it is not useful for identifying Group B streptococci.

85
Q

Which of the following organisms, if isolated from blood culture, can be potentially correlated to the presence of underlying intestinal cancer?

1 Clostridium perfringens
2 Staphylococcus lugdunensis
3 Burkholderia cepacia
4 Actinomyces naeslundii
5 Streptococcus bovis

A

Streptococcus bovis

S. bovis, isolated from blood cultures, may be associated with patients suffering from some form of intestinal cancer.
C. perfringens may, in fact, be normal flora in feces, and not usually associated with gastrointestinal malignancies.
S. lugdunensis, B. cepacia, and A. naeslundii are not usually isolated from feces, nor are they associated significantly with underlying malignancies.

86
Q

Ethylhydrocupreine hydrochloride is commonly used in the microbiology laboratory to confirm the presence of which of the following?

1 Enterococcus faecalis
2 Staphylococcus aureus
3 Streptococcus pneumoniae
4 Micrococcus sp
5 Lactobacillus sp

A

Streptococcus pneumoniae

Ethylhydrocupreine hydrochloride is also know as Optochin, as used in the “P” disk for identification of the pneumococcus. Optochin is only used on streptococci that are alpha-hemolytic and therefore must be separated into the potentially pathogenic S. pneumoniae and the commensal viridans streptococci.

87
Q

The aerococci and the enterococci resemble each other when observed in culture in the laboratory. The can be separated, however, because

1 The aerococci are PYR negative
2 The aerococci will not grow in 6.5% NaCl
3 The aerococci are bile esculine negative
4 The aerococci usually always appear as gram-positive cocci in tetrads
5 The aerococci are alpha-hemolytic

A

The aerococci usually always appear as gram-positive cocci in tetrads

Aerococci resemble enterococci in that they are PYR positive, grow in 6.5% NaCl broth, and some strains are bile esculin positive. Differentiating characteristics include the appearance of aerococci as tetrads on gram-stained preparations, in addtion to the fact that, aerococci are negative in the leucine aminopeptidase test (LAP), while the enterococci are positive in this test. The LAP test is included in many of the commercial kits used to identify the streptococci.

88
Q

A microbiology blood culture report is submitted from an automated instrument, and is posted in the patient’s chart. This report indicates the growth of Peptostreptococcus sp. after 4 days incubation. The most likely interpretation of this report will be

1 The patient clearly has a bacteremia due to Peptostreptococcus and should be treated with a beta-lactam or with metronidazole
2 The laboratory has probably introduced a contaminant into the blood bottle during the 4-days of processing and the blood culture should be repeated
3 This is a case of bacteremia due to an anaerobic organism, probably obtained from a food source, and the patient should be treated with a penicillin and with antitoxin
4 The organism in this case is probably a skin contaminant that was introduced into the bottle during specimen collection. No therapy is indicated, unless a repeat culture confirms the presence of the isolate, and the patient exhibits symptoms of septicemia
5 The blood culture should be repeated due to the potential intrinsic contamination of the blood culture bottle used to contain the blood

A

The organism in this case is probably a skin contaminant that was introduced into the bottle during specimen collection. No therapy is indicated, unless a repeat culture confirms the presence of the isolate, and the patient exhibits symptoms of septicemia

Peptostreptococcus sp . are normal flora on human skin. These flora are frequently are found as contaminants in blood bottles, due to inadequate preparation of the venipuncture site, by the phlebotomist. True infections with this organism are rare. The fact that this specimen took 4 days to appear, lends further support to the contamination possibility. In addition, this organism is not usually contracted through food, or food sources, and commercial blood bottles are rarely contaminated with any organism. The Peptostreptococci are, indeed, susceptible in vitro to the penicillins and to metronidazole but therapy, in this case, is probably unwarranted.

89
Q

Most anaerobic streptococci belong to which genus?

1 Helcococcus
2 Alloiococcus
3 Peptostreptococcus
4 Lactococcus
5 Stomatococcus

A

Peptostreptococcus

Most of the anaerobic cocci belong to the genus Peptostreptococcus. The other general on the list above are aerobic.

90
Q

One characteristic that sets the pathogenic Neisseria (N. gonorrhoeae and N. meningitidis) apart from the “nonpathogenic” Neisseria is

1 The pathogenic strains require blood-containing media, but the nonpathogenic strains will grow on media without blood
2 The pathogenic strains are nonpigmented on 5% sheep blood agar, and the nonpathogenic strains are not
3 The pathogenic strains exhibit strong metabolic activity against a variety of sugars, but the nonpathogenic strains do not
4 The pathogenic strains will grow under anaerobic conditions, and the nonpathogenic strains will not
5 The pathogenic strains produce beta-lactamases, but the nonpathogenic strains do not

A

The pathogenic strains require blood-containing media, but the nonpathogenic strains will grow on media without blood

N. gonorrhoeae and N. meningitidis are fastidious and require serum or blood in addition to other accessory growth factors, while the other Neisseria sp. will grow on nutrient agar devoid of blood. Pigmentation is usually not a definitive characteristic of the two pathogens. In addition, N. gonorrhoeae usually metabolizes only glucose, and N. meningitidis metabolized glucose, maltose, and no other sugars. Neither of these organisms are anaerobic, nor are other members of the same genus. Beta-lactamases can be produced by any or all species of the Neisseria.

91
Q

Which of the following conditions would be the most detrimental to N. meningitidis in a clinical specimen?

1 Collection on a calcium alginate swab containing charcoal and transported in Amies transport medium
2 Collection on a cotton swab and transported in a carbon-dioxide-containing transport container
3 Collection on a calcium alginate swab and refrigerated until the specimen can be hand-carried to the laboratory for culture
4 Collection on rayon swab and held at room temperature
5 Collection on cotton swabs, inserted into Stuarts transport medium at ambient temperature

A

Collection on a calcium alginate swab and refrigerated until the specimen can be hand-carried to the laboratory for culture

Both N. gonorrhoeae and N. meningitidis are extremely sensitive to storage at refrigerator temperature prior to culture. Regardless of the type of swab used, the specimen suspected of containing either of these organisms should be held and transported un-refrigerated, either at room temperature (ambient) or at 35° C.

92
Q

A direct Gram stain of clinical specimens may be helpful in many cases to confirm a diagnosis. When a diagnosis of gonorrhea is suspected, the direct Gram stain

1 Can be used effectively with specimens from men, but not from women
2 Can be used effectively with specimens from women, but not from men
3 Can be used effectively with specimens from both men and women
4 Should not be used in any case to evaluate a patient suspected of being infected
5 Can be used effectively only in immunocompromised individuals

A

Can be used effectively with specimens from men, but not from women

When used properly by trained personnel, the direct Gram stain is about 95% sensitive and 100% specific for the diagnosis of gonococcal urethritis in symptomatic men. In women, the sensitivity of endocervical smears in detecting gonorrhea falls to about 60% because of the normal vaginal flora that morphologically resemble N. gonorrhoeae.

93
Q

The specimen of choice for detecting and confirming the etiologic agent of gonorrhea in women is a

1 Vaginal specimen
2 Cervical specimen
3 Vaginal and cervical specimen
4 Cervical and pharyngeal specimen
5 Cervical and rectal specimen

A

Cervical and rectal specimen

Cervical and rectal cultures should be performed to diagnose infection in symptomatic and asymptomatic women, especially those known to be sexual contacts of infected men. In some instances, especially after antimicrobial therapy, rectal culture findings may be positive when endocervical culture findings are negative.

94
Q

Five colonial types of Neisseria gonorrhoeae have been described as each having unique characteristics. Which of the following statements is true regarding these colonial types?

1 All five types possess pili and can be isolated from infected individuals but the biochemical characteristics from each type are unique enough to distinguish them. Each colonial type is related to the virulence of the isolate.
2 Types 1 and 2 possess pili and are virulent and are the primary types isolated from infected individuals. After repeated subculture in the laboratory, types 1 and 2 usually revert to types 3, 4, and 5.
3 Types 3 and 4 are the common isolates from infected individuals and are easily recognized by their tiny size compared to the nonpathogenic types 1 and 2. Type 5 usually appears after storage and reculture.
4 Only type 5 is characteristically virulent because it is the only type that possesses pili thus providing a mechanism of attachment to the urethral epithelium. Types 1-4 represent the biochemically variant subspecies that can be recognized by their activity on specific carbohydrates.
5 Types 1-5 are simply colonial variants ranging from the largest (type 1) colony to the smallest (type 5) and are not related to virulence. All strains of N. gonorrhoeae, regardless of colonial type, eventually revert to the type 1 or 2 colony type after repeated transfer in the laboratory.

A

Types 1 and 2 possess pili and are virulent and are the primary types isolated from infected individuals. After repeated subculture in the laboratory, types 1 and 2 usually revert to types 3, 4, and 5.

The microbiologist can use a stereoscopic microscope to discern the five colony types of N. gonorrhoeae. Freshly isolated strains of N. gonorrhoeae consist of types 1 and 2. Type 1 is a small, raised, slightly viscid dewdrop colony whereas type 2 is small, raised, and friable. Types 1 and 2 may be maintained in the laboratory by selective transfer, but during nonselective transfer, types 1 and 2 revert to types 3, 4, or 5, which are larger and slightly convex and do not reflect incident light. Only types 1 and 2 possess pili and are virulent.

95
Q

The respiratory tract is colonized with a variety of organisms that morphologically resembles Neisseria meningitidis. Microscopic evaluation may also be difficult for the inexperienced laboratorian. Which organism below could most likely be confused with Neisseria meningitidis when a specimen is taken from chocolate agar?

1 Neisseria elongata
2 Leminorella grimontii
3 Moraxella catarrhalis
4 Stenotrophomonas maltophilia
5 Porphyromonas sp

A

Moraxella catarrhalis

Moraxella catarrhalis and Neisseria sp. can be easily confused morphologically and microscopically since both are Gram negative diplococci and are oxidase positive. Moraxella , however, are biochemically inert and are DNAse positive, unlike N. meningitidis. N. elongata is a rod, as are Leminorella grimontii, Stenotrophomonas maltophilia, and Porphyromonas sp.

96
Q

In which population group below would one most likely find the highest attack rate for gonorrhea in the United States?

1 Adult men who have sex with men
2 Female sex workers >25 years old
3 Immunocompromised adults
4 Middle school adolescents
5 Young adults 15-24 years old

A

Young adults 15-24 years old

Gonorrhea is the most commonly reported infectious disease in the United States. In the U.S., there are about 1,000,000 cases reported each year and probably the same number that go unreported. The highest rates are in young adults, aged 15-24 years.

97
Q

Several gonococcal vaccines have been evaluated for efficacy in humans, but none have yet provided sufficient protection. Due to the pathogenic mechanism of Neisseria gonorrhoeae , the most likely antigen from which to base vaccine production for this organism would be

1 Cell wall peptidoglycans
2 Membrane lipopolysaccharides
3 Extracellular capsules
4 Pili
5 Teichoic acid

A

Pili

Efforts to develop a vaccine for gonorrhea have centered on the use of pili or protein I as the immunogen. Pilus antigens are attractive vaccine candidates because development of antibodies to the pilus might block the initial attachment of the organism to epithelial cells. One major problem with using pili, however, has been their antigenic diversity and the need to develop a polyvalent vaccine that would provide protection against a variety of pilus types.

98
Q

The Waterhouse-Friderichsen syndrome is associated with which organism?

1 Moraxella catarrhalis
2 Neisseria meningitidis
3 Stenotrophomonas maltophilia
4 Kingella indologenes
5 Yersinia enterocolitica

A

Neisseria meningitidis

Sometimes, meningococcemia may take a chronic, moderate, or fulminant course. In the fulminant form, known as Waterhouse-Friderichsen syndrome, disseminated intravascular coagulation (DIC) develops with subsequent hemorrhaging into the skin, the adrenal glands, and other internal organs. The petechiae on the skin coalesce to form large ecchymotic lesions. The condition progresses rapidly, and without appropriate therapy, the patient may die within a few hours.

99
Q

Which of the following are true concerning human infection with Streptococcus pneumoniae?

1 S. pneumoniae is one of the most common causes of bacterial pneumonia
2 The mortality associated with pneumococcal pneumonia in hospitalized patients is 20-30%
3 The incidence of pneumococcal pneumonia decreases sharply after 50 years of age
4 While other streptococcal species may be recovered from the pharyngeal swabs of healthy people, the detection of S. pneumoniae is always associated with disease
5 Due to increasing selection pressure caused by widespread antibiotic usage, the majority of pneumococcal isolates are no longer sensitive to penicillin

A

S. pneumoniae is one of the most common causes of bacterial pneumonia

S. pneumoniae remains one of the most common causes of bacterial pneumonia. However, the mortality associated with S. pneumoniae pneumonia in hospitalized patients is generally 5-10% (down from 20-30% in the pre-antibiotic era). Both the incidence and mortality associated with S. pneumoniae pneumonia increase after 50 years of age, and the presence of other chronic diseases may predispose the individual to a worsened course or a higher risk of mortality. Normal carriage of S. pneumoniae is relatively common, even in healthy individuals. Penicillin remains the treatment of choice, as most pneumococcal strains are still sensitive to this agent.

100
Q

Case
A 38-year-old man presents with nausea, vomiting, diarrhea, and abdominal cramping for the past 2 hours. The patient reveals that he had consumed meat that had been left uncovered for a night. Laboratory examination revealed that the meat was contaminated with Gram-positive circular, clustered organisms. Fairly large, yellow-colored colonies were formed when the organism was grown on mannitol agar media at 40°C. The organism showed positive results for coagulase enzyme activity and showed resistance to penicillin and methicillin antibiotics.

What organism was the causative agent of the food poisoning?

1 Clostridium perfringens
2 Enterococcus faecalis
3 Escherichia coli
4 Staphylococcus aureus
5 Salmonella species

A

Staphylococcus aureus

The causative agent of food poisoning in this case is Staphylococcus aureus. S. aureus is a Gram-positive, aerobic, circular (cocci) organism that appears to be in pairs or in grape-like clusters on microscopic examination. S. aureus is found as normal human body flora colonized in the nasal passages. The organism is pathogenic, pertaining to its ability to produce and release toxins. It is resistant to high salt concentrations, especially sodium chloride (NaCl) and grows on mannitol salt agar forming large, yellow-colored colonies. S. aureus grows well in a temperature range of 15 to 45°C. The majority of the strains of S. aureus show positive results for coagulase test, but it is not a differential diagnostic test to detect the organism, as few strains of S. epidermidis also show positive results for the test. S. aureus is resistant to penicillin and methicillin. Individuals with S. aureus food poisoning have loss of fluid due to diarrhea; therefore, they are advised to drink plenty of water and electrolyte solutions. The preventive measures include proper washing of hands and maintaining good hygiene when preparing or handling foods.
Clostridium perfringens is a Gram-positive, anaerobic, spore-forming bacilli (rod shaped). C. perfringens is commonly found as a soil microorganism. It is also found in the feces of infected persons and thus becomes a source of infection. C. perfringens food poisoning is due to the ingestion of the organism and is associated with intense abdominal cramps and diarrhea. The diagnostic tests include stool examination to detect the toxin or the organism in the infected person’s feces. The organism can grow in a wide range of temperatures, and it is cultured in anaerobic liquid media in laboratories. There is no specific treatment for C. perfringens food poisoning, but the preventive measures include cooking food at high temperatures, rapid cooling of cooked foods, and hygienic handling of foods.
Enterococcus faecalis, previously known as Streptococcus faecalis, is a Gram-positive, circular, facultative anaerobe. It is a normal inhabitant of the intestinal tract and female genital tract. It may cause endocarditis, urinary tract infections, bacteremia, wound infections, catheter-related infections, and intra-abdominal/pelvic infections.
Escherichia coli are a common flora of the human body and are generally found in the gut of humans. E. coli are Gram-negative, non-spore forming bacilli. Most strains of E. coli are non-pathogenic, but strains called enterohaemorrhagic E. coli (EHEC) are responsible for foodborne illness due to the toxins they release. The symptoms include abdominal cramps and diarrhea, which may lead to hemorrhagic colitis. The organism grows in a temperature range of 7 to 50°C, but the optimum temperature at which the organism grows best is 37°C. Diagnosis of the infection is generally by microscopic examination of the stool from the infected person. Preventive measures include practicing hygiene when preparing food and using boiled and cooled water for consumption.
Salmonella species are non-spore forming, Gram-negative bacilli. Salmonella are found in the intestinal tract of humans and other warm and cold-blooded animals. They are responsible for causing acute gastroenteritis as a result of foodborne ingestion or intoxication. They are transmitted to the humans via contaminated water and food. Salmonella typhi and Salmonella paratyphi are responsible for typhoid and typhoid-like fever in humans. Measures to prevent infection include hygienic practices when handling food.

101
Q

Case
A 20-year-old male college student presents with fever, chills, malaise, headache, photophobia, confusion, and numerous petechiae on his extremities and trunk. On examination, he has positive Brudzinski and Kernig’s signs. A CT scan of the head is within reference limits. A lumbar puncture reveals increased leukocytes, particularly polymorphonuclear neutrophils, increased protein, and decreased glucose levels. Gram staining of the CSF reveals Gram-negative cocci in pairs.

What is the most likely causative agent for the infection in this patient?

1 Bordetella pertussis
2 Francisella tularensis
3 Haemophilus influenzae
4 Neisseria meningitidis
5 Yersinia pestis

A

Neisseria meningitidis

Meningitis can be caused by bacteria, viruses, and fungi. Neisseria meningitidis is the most likely causative agent based on the Gram staining finding and the age and clinical presentation of the patient. N. meningitidis is a Gram-negative diplococci usually found as normal flora of the upper respiratory tract that cause symptoms such as skin lesions, fever, malaise, and headache. N. meningitidis is the most common cause of acute bacterial meningitis in patients ages 18-60, and two classic signs of acute bacterial meningitis are positive Kernig’s and Brudzinski signs. Kernig’s sign is the inability to fully extend the knee when the hip is flexed to 90° due to severe hamstring stiffness. Brudzinski sign is flexion of the hips and knees with passive flexion of the neck due to neck stiffness. Treatment for bacterial meningitis often involves ceftriaxone and vancomycin due to presence of increasing ceftriaxone resistance in the US.
Bordetella is a Gram-negative rod that causes whooping cough in humans. The organisms colonize in the upper respiratory tract and result in fever, malaise, and a cough characterized by an inspiratory gasp (whoop). This infection is highly contagious and is spread by coughing and nasal drops. This patient does not have a cough.
Francisella is a Gram-negative rod responsible for tularemia (rabbit fever). The reservoir is in rabbits, and it is transmitted by ticks. The bacteria multiply at the site of penetration in the skin and are transported to regional lymph nodes. Patients have a high fever, chills, headache, and other symptoms similar to the plague.
Haemophilus influenzae can cause meningitidis in infants and children 6 months to 6 years, but the incidence of meningitis due to H. influenzae has declined significantly since the mid-1980s as a result of the widespread use of vaccination against this organism. The age of this patient rules out H. influenzae as the causative agent.
Yersinia is a Gram-negative rod that causes bubonic plague in humans. Yersinia is transmitted by fleas from rodents to humans. This patient does not have bubonic plague.

102
Q

A 5-year-old boy presents with papular and pustular lesions on his face. A serous, honey-colored fluid exudes from the lesions. You suspect impetigo. A Gram stain of the pus reveals many neutrophils and Gram-positive cocci. If you send the pus to the laboratory to make a microbiologic diagnosis, what would be the most likely laboratory result?

1 Catalase-positive, coagulase-positive, beta-hemolytic colonies
2 Catalase-positive, coagulase-negative, non-hemolytic colonies
3 Catalase-negative, beta-hemolytic, bacitracin-resistant colonies
4 Catalase-negative, alpha-hemolytic, Optochin-inhibited colonies
5 Catalase-negative, alpha-hemolytic colonies, not inhibited by Optochin

A

Catalase-positive, coagulase-positive, beta-hemolytic colonies

Impetigo is a superficial contagious skin infection with 2 types: bullous and non-bullous. Bullous impetigo is caused most exclusively by Staphylococcus aureus. Non-bullous impetigo is caused by S. aureus or streptococcus pyogenes (Group A beta-hemolytic streptococci-GABHS); both may also be present.
Staphylococci are spherical Gram-positive cocci arranged in irregular grape-like clusters. S. aureus is catalase positive, coagulase positive, and produces beta-hemolytic colonies. S. aureus is distinguished from the other staphylococcus primarily by coagulase production; it usually ferments mannitol and hemolyzes blood, whereas the others do not.
Non-bullous impetigo usually occurs in children, whereas bullous impetigo may occur at all ages, but it is more common in infants. Impetigo develops at sites where there is minor skin injury or trauma, wherein the skin integrity is lost. It starts as a pustule and forms a blister, which bursts to exude a serous honey colored fluid that crusts. Diagnosis is by skin swab for culture and sensitivity. Treatment with topical application of mupirocin is effective in mild to moderate cases. Oral systemic treatment is required in extensive lesions with amoxicillin, dicloxacillin, or erythromycin. Clindamycin, when taken 300mg orally every 8 hours for 5-7 days, is also effective.
Staphylococcus epidermidis is catalase-positive, coagulase-negative, and produces non-hemolytic colonies. It is part of normal human flora on the skin and mucous membranes but can cause infections of intravenous catheters and prosthetic implants.
Streptococci are spherical Gram-positive cocci usually arranged in chains or pairs. All streptococci are catalase-negative, whereas staphylococci are catalase-positive. Alpha-hemolytic streptococci form a green zone around their colonies as a result of incomplete lysis of red blood cells in the agar. Beta-hemolytic streptococci form a clear zone around their colonies since complete lysis of red cells occurs. Beta hemolysis is due to the production of enzymes (hemolysins) called streptolysin O and streptolysin S. Beta-hemolytic streptococci are arranged into groups A-U (known as Lancefield groups) on the basis of antigenic differences in C carbohydrate. Group A beta hemolytic streptococcus, also known as S. pyogenes, also causes impetigo.
Streptococcus pneumoniae are Gram-positive lancet-shaped cocci arranged in pairs (diplococci) or short chains. On blood agar, they produce alpha-hemolysis.
Streptococcus viridans is catalase-negative, coagulase-negative, causes alpha-hemolysis on blood agar, and is not inhibited by Optochin.

103
Q

Streptococcus pneumoniae vaccine contains which of the following?

1 The capsular polysaccharide of the 23 most prevalent types
2 A toxoid
3 Killed Streptococcus pneumoniae
4 An antigen purified from the organism
5 Attenuated strains of Streptococcus pneumoniae

A

The capsular polysaccharide of the 23 most prevalent types

Streptococcus pneumoniae vaccine contains the capsular polysaccharides of the 23 most prevalent types. The vaccine is recommended for persons over 60 years of age and patients of any age with chronic diseases such as diabetes and cirrhosis or with compromised spleen function or splenectomy.

104
Q

Neisseria gonorrhoeae is the causative agent of gonorrhea infections in humans. Which of the following most properly describes the characteristics of gonococci?

1 Gram-negative diplococci which are non-motile and spore-forming
2 Gram-positive diplococci which are motile and non-spore-forming
3 Gram-negative streptococci which are non-motile and non-spore-forming
4 Gram-positive diplococci which are highly resistant to drying
5 Gram-negative diplococci which are non-motile and non-spore-forming

A

Gram-negative diplococci which are non-motile and non-spore-forming

Neisseria gonorrhoeae (or gonococci) are gram-negative diplococci which are non-motile and non-spore-forming, and do not tolerate drying. Gonococcal growth is optimal between 35-37o C in an atmosphere containing 5% CO2. A complex growth medium is required, and selective media containing antibiotics are often used for culturing sites colonized by less fastidious competing organisms.

105
Q

Case
A 5-year-old boy has three honey-colored crusted lesions with surrounding erythema on his legs. The swabs taken from the lesions were sent to the microbiology laboratory. The results show yellow colonies grown on blood agar with hemolysis. The colonies are coagulase-positive and mannitol-positive.

What is the most likely diagnosis?

1 Molluscum contagiosum
2 Varicella zoster
3 Impetigo
4 Herpes zoster
5 Erythema infectiosum

A

Impetigo

The case described here is impetigo, a superficial skin infection common in children. The arms, legs, and face are more susceptible to impetigo than unexposed areas. Lesions vary from pea-sized vesicopustules to large ringworm-like lesions. They can start as maculopapules and rapidly progress to vesicopustules. The pustules enlarge and form a thick crust. Impetigo is classically described as honey-colored or golden crusted lesions.
The most frequent cause is Staphylococcus aureus, and the second is Streptococcus pyogenes. Staphylococcus aureus produces yellow-white colonies on blood agar with hemolysis and S. aureus is coagulase- and mannitol-positive, and these tests are used in the lab to differentiate S. aureus from other staphylococci. Streptococcus pyogenes is not coagulase and mannitol positive.
Varicella zoster can present with crusted lesions at some point in its clinical course, but the distribution given in the clinical picture does not match a varicella infection. Varicella causes a generalized rash including the face, trunk, and extremities.
Herpes zoster is typically unilateral and distributed in a single dermatome. This patient has a rash on both legs, making herpes zoster unlikely.
Molluscum contagiosum presents as flesh-colored umbilicated papules, not crusted lesions.
Erythema infectiosum (slapped-cheek, fifth disease) presents with an erythematous rash on the cheeks with surrounding pallor.

106
Q

Case
A 32-year-old woman is brought to the emergency room with sudden onset of fever, headache, sore throat, profuse watery diarrhea, vomiting, and lethargy; symptoms started in the morning. On physical examination, she is slightly confused; her temperature is 39°C (103°F), her blood pressure is 100/50 mm Hg, and she has diffuse sunburn like an erythematous rash. When the emergency doctor is trying to find out if she is pregnant, she mentions that her period started 2 days ago.

What is the most likely diagnosis?

1 Scarlet fever
2 Rocky Mountain spotted fever
3 Meningococcemia
4 Toxic shock syndrome
5 Food poisoning

A

Toxic shock syndrome

This is a typical case presentation of toxic shock syndrome (TSS). The patient is a young woman; she is menstruating, and the disease had a sudden abrupt onset with rapid deterioration.
TSS is caused by toxic shock syndrome toxin-1 (TSST-1) produced by Staphylococcus aureus. TSST-1 is a pyrogenic exotoxin, which causes fever, multiple organ dysfunction, rash, hypotension, syncope, and shock. TSS can be seen in menstruating women who use tampons, individuals with wound infections, or patients with nasal packing to stop bleeding from the nose.
Scarlet fever also presents with sore throat and a fine erythematous rash, but diarrhea and shock are unlikely.
Meningococcemia is a life-threatening condition and should be suspected in every case presenting with rash, fever, and headache. Meningococcal rash is more petechial, and it progresses to gangrenous lesions if not treated.
Rocky Mountain spotted fever also presents with fever and a headache with a sudden onset, but the typical rash appears 2-6 days later. It initially starts on the hands and feet, then moves to the trunk. It is a rickettsial disease transmitted by ticks, from dogs, and rodents.
Food poisoning can have acute onset with fever, but it is very unlikely to present with a rash.

107
Q

A 15-year-old male comes into your office. He has several furuncles on his neck, with pus exuding from the lesions. A Gram stain of the pus reveals many neutrophils and gram-positive cocci in grape-like clusters. Which one of the following organisms is the most likely cause of this disease?

1 Streptococcus pyogenes (group A beta-hemolytic)
2 Staphylococcus epidermidis
3 Staphylococcus aureus
4 Peptostreptococcus
5 Streptococcus pneumoniae

A

Staphylococcus aureus

Streptococci are spherical gram-positive cocci usually arranged in chains or pairs. All streptococci are catalase-negative; whereas staphylococci are catalase-positive. Alpha-hemolytic streptococci form a green zone around their colonies as a result of incomplete lysis of red blood cells in the agar. Beta-hemolytic streptococci form a clear zone around their colonies, since complete lysis of red cells occurs. Beta hemolysis is due to the production of enzymes (hemolysins) called streptolysin O and streptolysin S.
Streptococcus pyogenes (group A beta-hemolytic streptococcus) causes 3 types of diseases: 1) pyogenic diseases, such as pharyngitis and cellulitis, 2) toxigenic diseases, such as scarlet fever and toxic shock syndrome, and 3) immunologic diseases, such as rheumatic fever and acute glomerulonephritis.
Streptococcus pneumoniae are gram-positive lancet-shaped cocci arranged in pairs (diplococci) or short chains. On blood agar they produce alpha-hemolysis. Virulence factors of pneumococci are polysaccharide capsules. Pneumococci cause pneumonia, bacteremia, meningitis, and infections of the upper respiratory tract, such as otitis and sinusitis. Mortality rate is high in elderly, immunocompromised (especially splenectomized), or debilitated patients. They should be immunized with the polyvalent polysaccharide vaccine.
Peptostreptococci grow under anaerobic or microaerophilic conditions and produce variable hemolysis. Peptostreptococci are members of the normal flora of the gut and female genital tract and participate in mixed anaerobic infections of the abdomen, pelvis, lungs, and brain.
Staphylococci are spherical gram-positive cocci arranged in irregular grape-like clusters. S. Aureus is distinguished from the others primarily by coagulase production; it usually ferments mannitol and hemolyzes blood, whereas the others do not. S. Aureus is often found in the nose and sometimes on the skin. Staphylococcus aureus causes inflammatory and toxin-mediated diseases. Inflammatory diseases include: 1) skin infections, including impetigo, furuncles, carbuncles, paronychia, cellulitis, surgical wound infections, eyelid infections, and postpartum breast infections; 2) septicemia (sepsis) can originate from any localized lesion, especially wound infection, or as a result of intravenous drug abuse; 3) endocarditis on normal or prosthetic heart valves; 4) osteomyelitis and arthritis, either hematogenous or traumatic; 5) pneumonia in postoperative patients or following viral respiratory infection, especially influenza, 6) abscesses (metastatic) in any organ.
Toxin-mediated diseases include 1) food poisoning characterized by vomiting being more prominent than diarrhea and short incubation period (1-8 hours); 2) toxic shock syndrome, which includes fever, hypotension, a rash that goes on to desquamate, and multisystem involvement; and 3) scalded skin syndrome.
Staphylococcus epidermidis is part of normal human flora on the skin and mucous membranes but can cause infections of intravenous catheters and prosthetic implants.

108
Q

A 15-year-old male comes into your office. He has several furuncles on his neck, with pus exuding from the lesions. A Gram stain of the pus reveals many neutrophils and gram-positive cocci in grape-like clusters. If you send pus to laboratory to make a microbiologic diagnosis, which of the following would most likely be the laboratory result?

1 Catalase-positive, coagulase-positive, beta-hemolytic colonies
2 Catalase-positive, coagulase-negative, nonhemolytic colonies
3 Catalase-negative, beta-hemolytic, bacitracin-sensitive colonies
4 Catalase-negative, alpha-hemolytic, Optochin-inhibited colonies
5 Catalase-negative, alpha-hemolytic colonies, not inhibited by Optochin

A

Catalase-positive, coagulase-positive, beta-hemolytic colonies

Refer to the table. Streptococci are spherical gram-positive cocci, usually arranged in chains or pairs. All streptococci are catalase-negative; whereas, staphylococci are catalase-positive. Alpha-hemolytic streptococci form a green zone around their colonies as a result of incomplete lysis of red blood cells in the agar. Beta-hemolytic streptococci form a clear zone around their colonies, since complete lysis of red cells occurs. Beta hemolysis is due to the production of enzymes (hemolysins) called streptolysin O and streptolysin S. Beta-hemolytic streptococci are arranged into groups A-U (known as Lancefield groups) on the basis of antigenic differences in C carbohydrate.
Streptococcus pneumoniae are gram-positive lancet-shaped cocci arranged in pairs (diplococci) or short chains. On blood agar they produce alpha-hemolysis.
Staphylococci are spherical gram-positive cocci arranged in irregular grape-like clusters. S. Aureus is distinguished from the others primarily by coagulase production; it usually ferments mannitol and hemolyzes blood; whereas, the others do not. Staphylococcus epidermidis is part of normal human flora on the skin and mucous membranes but can cause infections of intravenous catheters and prosthetic implants.
Staphylococci of medical importance
Species Catalase
production Coagulase
production Typical
hemolysis Important features
Staphylococcus
aureus positive positive beta ferments mannitol
Staphylococcus
epidermis positive negative none sensitive to novobiocin
Staphylococcus
saprophyticus positive negative none resistant to novobiocin
Streptococci of medical importance
Species Catalase
production Lancefield
group Typical
hemolysis Diagnostic features
Streptococcus
pyogenes negative A beta bacitracin-sensitive
Streptococcus
agalactiae negative B beta bacitracin-resistant;
hippurate hydrolyzed
Streptococcus
faecalies
(Enterococcus
faecalis) negative C alpha or
beta or
none
growth in 6,5% NaCl
Streptococcus
bovis negative D alpha or
beta or
none
no growth in 6,5%
NaCl
Streptococcus
pneumoniae negative - alpha
bile-soluble; inhibited
by optochin
Streptococcus
viridans negative - alpha
not bile-solube; not
inhibited by optochin

109
Q

Case
A 10-year-old boy is brought to the clinic by his mother. She noted that his face is swollen, and he told her that his urine was cloudy and reddish. He has a history of falling and abrading the skin of his right thigh 2 weeks ago. The next day, the skin became red, hot, and tender; the infection was treated with a topical antibiotic ointment. The cellulitis gradually healed.

What organism is the most likely cause of this patient’s disease?

1 Streptococcus pyogenes (group A beta-hemolytic)
2 Streptococcus agalactiae (group B)
3 Enterococcus faecalis
4 Peptostreptococcus
5 Streptococcus pneumoniae

A

Streptococcus pyogenes (group A beta-hemolytic)

Streptococcus pyogenes (group A beta-hemolytic) cause three types of diseases:
pyogenic diseases, such as pharyngitis and cellulitis
toxigenic diseases, such as scarlet fever and toxic shock syndrome
immunologic diseases, such as rheumatic fever and acute glomerulonephritis. Glomerulonephritis occurs especially following skin infections.
Streptococcus pneumoniae are gram-positive lancet-shaped cocci arranged in pairs (diplococci) or short chains. On blood agar, they produce alpha-hemolysis. Virulence factors of Pneumococci are polysaccharide capsules. Pneumococci cause pneumonia, bacteremia, meningitis, and infections of the upper respiratory tract, such as otitis and sinusitis. Mortality rate is high in elderly, immunocompromised (especially splenectomized), and/or debilitated patients. They should be immunized with the polyvalent polysaccharide vaccine.
Peptostreptococci grows under anaerobic or microaerophilic conditions and produces variable hemolysis. Peptostreptococci are members of the normal flora of the gut and female genital tract and participate in mixed anaerobic infections of the abdomen, pelvis, lungs, and brain.
Streptococcus agalactiae (group B streptococcus) colonize the genital tract of some women and can cause neonatal meningitis and sepsis. They are usually bacitracin-resistant.
Enterococcus faecalis (group D streptococcus), formerly known as Streptococcus faecalis, are part of the normal flora in the gut. They can cause urinary, biliary, and cardiovascular infections.

110
Q

3 hours after a delicious Thanksgiving dinner of barley soup, roast turkey, stuffing, sweet potato, green beans, cranberry sauce, pumpkin pie topped with whipped cream, and ice-cream, the Jones family of 5 experience vomiting and diarrhea. Which one of the following organisms is the most likely to be involved?

1 Salmonella enteritidis
2 Bacillus cereus
3 Clostridium perfringens
4 Staphylococcus aureus
5 Vibrio parahaemolyticus

A

Staphylococcus aureus

Bacillus cereus cause 2 syndromes: (1) one involves a short incubation period with nausea and vomiting and is similar to staphylococcal food poisoning; (2) the other involves a long incubation period (18 hours) with watery, non-bloody diarrhea and resembles clostridial gastroenteritis.
Clostridium perfringens cause food poisoning with an 8- to 16-hour incubation period and is characterized by watery diarrhea with cramps and little vomiting. It resolves in 24 hours.
Staphylococcus aureus cause food poisoning characterized by vomiting being more prominent than diarrhea. The disease is due to ingestion of enterotoxin, which is pre-formed in foods and hence has a short incubation period (1-8 hours).
Vibrio parahaemolyticus are a marine organism transmitted by contaminated seafood, especially when raw fish is eaten. The clinical picture varies from mild to quite severe watery diarrhea, nausea and vomiting, abdominal cramps, and fever. The illness is self-limited, lasting about 3 days.
Salmonella enteritidiscause enterocolitis after an incubation period of 6-48 hours. The disease begins with nausea and vomiting and then progresses to abdominal pain and diarrhea, which can vary from mild to severe, with or without blood. Usually, the disease lasts a few days and is self-limited.
Refer to the table for additional information.
Bacterial diarrheal diseases transmitted by foods
Bacterium Typical food Disease
Gram-positive cocci
Staphylococcus aureus Custard-filled pastries; potato, egg, or tuna fish salad Food poisoning, especially vomiting
Gram positive rods
Bacillus cereus Reheated rice Diarrhea
Clostridium perfringens Cooked meat, stew, gravy Diarrhea
Gram negative rods
Escherichia coli Various foods and water Diarrhea
Escherichia coli
O157:H7 strain Undercooked meat Hemorrhagic colitis
Salmonella enteritidis Poultry, meats, and eggs Diarrhea
Shigella species Various foods and water Diarrhea (dysentery)
Vibrio cholerae Various foods, e.g. seafood, and water Diarrhea
Vibrio parahaemolyticus Seafood Diarrhea
Campylobacter jejuni Various foods Diarrhea
Yersinia enterocolitica Various foods Diarrhea

111
Q

A 25-year-old man comes into your office. He was well until yesterday when he experienced dysuria, accompanied by yellowish urethral discharge. He is single and he admits that he had intercourse with a sex worker five nights ago. You perform a Gram stain on the specimen of the discharge and see gram-negative diplococci within neutrophils. Which one of the following is the most likely cause of the infection?

1 Neisseria gonorrhoeae
2 Haemophilus ducreyi
3 Treponema pallidum
4 Chlamydia trachomatis
5 Ureaplasma urealyticum

A

Neisseria gonorrhoeae

Neisseriae are gram-negative, oxidase-positive cocci that resemble paired kidney beans. N. gonorrhoeae causes gonorrhea, neonatal conjunctivitis and pelvic inflammatory disease. Gonorrhea in men is characterized primarily by urethritis accompanied by dysuria and a purulent discharge. In women, infection is located primarily in the endocervix, causing a purulent vaginal discharge and intermenstrual bleeding (cervicitis). The most frequent complication in women is ascending infection of the uterine tubes (salpingitis, PID), which can result in sterility or ectopic pregnancy. Disseminated infections commonly manifest as arthritis, tenosynovitis or pustules. Other infected sites include anorectal area (in women and homosexual men) and the throat.
In men, the finding of gram-negative diplococci within PMNs in a sample of urethral discharge is sufficient for diagnosis. In women, the use of the Gram stain alone can be difficult to interpret, so cultures should be done.
Neisseria gonorrhoeae
grows best on Thayer-Martin medium, which is a chocolate agar containing antibiotics (vancomycin, colistin, trimethoprim and nystatin) to suppress the normal flora and are incubated at 37° C in a 5% CO2 atmosphere. The finding of an oxidase-positive colony composed of gram-negative diplococci is sufficient to diagnose Neisseria. The differentiation between N. meningitidis and N. gonorrhoeae is made on the basis of sugar fermentation: meningococci ferment maltose, whereas gonococci do not (both organisms ferment glucose).
Haemophilus ducreyi
is small gram-negative rod that causes chancroid - sexually transmitted disease. The disease begins with penile lesions, which are painful: nonindurated (soft) ulcers: and local lymphadenitis.
Treponema pallidum
is a spirochete - thin-walled, flexible, spiral, motile rod. They are so thin that they are seen only by darkfield microscopy, silver impregnation, or immunofluorescence.
Chlamydiae
are obligate intracellular bacteria; the Gram stain is not useful. Chlamydia trachomatis exists in more than 15 immunotypes (A-L). Types A-C cause trachoma. Types D-K cause genital tract infections. In men, it is a common cause of nongonococcal urethritis, which may progress to epididymitis, prostatitis or proctitis. In women, cervicitis develops and may progress to salpingitis and pelvic inflammatory disease. Types L1-L3 cause lymphogranuloma venereum.
Ureaplasma urealyticum
may cause approximately 20% of cases of nongonococcal urethritis. Ureaplasma is the smallest free-living organism, without a cell wall. Therefore, it stains poorly with Gram’s stain.

112
Q

A 26-year-old woman is brought to the clinic because of fever and abdominal pain. She was well until two days ago, when mild, crampy, left lower quadrant pain began, accompanied by malaise and anorexia. She has noticed an increased amount of yellowish, non-bloody discharge, with no odor. Her last menstrual period was ten days ago.
T 39° C, BP 120/75, P 92. On physical exam: Cervix: inflamed with a small amount of purulent discharge at os. Motion tenderness elicited. Uterus: normal size, no tenderness or masses. Adnexa: very tender on left.
Hct 39%, WBC 16.000. Culdocentesis yielded 20 ml of bloody, foul smelling, purulent fluid that was sent for aerobic and anaerobic culture. Gram stain of the fluid revealed gram-negative diplococci. Which one of the following is the most likely cause of the infection?

1 Neisseria gonorrhoeae
2 Treponema pallidum
3 Gardnerella vaginalis
4 Chlamydia trachomatis
5 Haemophilus ducreyi

A

Neisseria gonorrhoeae

Haemophilus ducreyi is small gram-negative rod that causes chancroid - sexually transmitted disease. The disease begins with penile lesions, which are painful: non-indurated (soft) ulcers: and local lymphadenitis.
Gardnerella vaginalis is a facultative gram-variable rod associated with bacterial vaginosis, characterized by a malodorous vaginal discharge and “clue cells” (vaginal epithelial cells covered with bacteria).
Chlamydiae are obligate intracellular bacteria; the Gram stain is not useful. Chlamydia trachomatis exists in more than 15 immunotypes (A-L). Types A-C cause trachoma. Types D-K cause genital tract infections. In men, it is a common cause of nongonococcal urethritis, which may progress to epididymitis, prostatitis or proctitis. In women, cervicitis develops and may progress to salpingitis and pelvic inflammatory disease. Types L1-L3 cause lymphogranuloma venereum.
Treponema pallidum is a spirochete - thin-walled, flexible, spiral, motile rod. They are so thin that they are seen only by darkfield microscopy, silver impregnation, or immunofluorescence.
Neisseriae are gram-negative, oxidase-positive cocci that resemble paired kidney beans. N. gonorrhoeae causes gonorrhea, neonatal conjunctivitis and pelvic inflammatory disease. Gonorrhea in men is characterized primarily by urethritis accompanied by dysuria and a purulent discharge. In women, infection is located primarily in the endocervix, causing a purulent vaginal discharge and intermenstrual bleeding (cervicitis). The most frequent complication in women is ascending infection of the uterine tubes (salpingitis, PID), which can result in sterility or ectopic pregnancy. Disseminated infections commonly manifest as arthritis, tenosynovitis or pustules. Other infected sites include anorectal area (in women and homosexual men) and the throat.
In men, the finding of gram-negative diplococci within PMNs in a sample of urethral discharge is sufficient for diagnosis. In women, the use of the Gram stain alone can be difficult to interpret, so cultures should be done.
Neisseria gonorrhoeae grows best on Thayer-Martin medium, which is a chocolate agar containing antibiotics (vancomycin, colistin, trimethoprim and nystatin) to suppress the normal flora, and is incubated at 37° C an a 5% CO2 atmosphere. The finding of an oxidase-positive colony composed of gram-negative diplococci is sufficient to diagnose Neisseria. The differentiation between N. meningitidis and N. gonorrhoeae is made on the basis of sugar fermentation: meningococci ferment maltose, whereas gonococci do not (both organisms ferment glucose).

113
Q

A 26-year-old woman is brought to the clinic because of fever and abdominal pain. She was well until two days ago, when mild, crampy, left lower quadrant pain began, accompanied by malaise and anorexia. She has noticed an increased amount of yellowish, non-bloody discharge, with no odor. Her last menstrual period was ten days ago.
T 39° C, BP 120/75, P 92. On physical exam: Cervix: inflamed with a small amount of purulent discharge at os. Motion tenderness elicited. Uterus: normal size, no tenderness or masses. Adnexa: very tender on left.
Hct 39%, WBC 16.000. Culdocentesis yielded 20 ml of bloody, foul smelling, purulent fluid that was sent for aerobic and anaerobic culture. Gram stain of the fluid revealed gram-negative diplococci. You suspect PID caused by N. gonorrhoeae. If you send a specimen for the culture, the organism will most likely grow on

1 Thayer-Martin medium
2 MacConkey’s agar
3 Chocolate agar enriched with factor X
4 EMB agar
5 Chocolate agar enriched with factor X and factor V

A

Thayer-Martin medium

N. gonorrhoeae are gram negative, oxidase-positive cocci that resemble paired kidney beans. Neisseria gonorrhoeae grows best on Thayer-Martin medium, which is a chocolate agar containing antibiotics (vancomycin, colistin, trimethoprim and nystatin) to suppress the normal flora and is incubated at 37° C in a 5% CO2 atmosphere. The finding of an oxidase-positive colony composed of gram-negative diplococci is sufficient to diagnose Neisseria. The differentiation between N. meningitidis and N. gonorrhoeae is made on the basis of sugar fermentation: meningococci ferment maltose, whereas gonococci do not (both organisms ferment glucose).
Haemophilus influenzae requires addition of two components, factor X (heme) and factor V (NAD), for adequate energy production.
Haemophilus ducreyi requires heated (chocolate) blood agar supplemented with factor X (heme), but unlike Haemophilus influenzae, does not require factor V.
MacConkey’s and EMB (eosin-methylene blue) agar are selective differential agar, used in the identification of the gram-negative rods. Their differential ability is based on lactose fermentation, which is the most important metabolic criterion. On this medium, the non-lactose fermenters, e.g. Salmonella and Shigella, form colorless colonies, whereas the lactose fermenters form colored colonies. The selective effect of these media in suppressing unwanted gram-positive organisms is exerted by bile salts or bacteriostatic dyes in the agar.

114
Q

A 26-year-old woman is brought to the clinic because of fever and abdominal pain. She was well until two days ago, when mild, crampy, left lower quadrant pain began, accompanied by malaise and anorexia. She has noticed an increased amount of yellowish, non-bloody discharge, with no odor. Her last menstrual period was ten days ago.
T 39° C, BP 120/75, P 92. On physical exam: Cervix: inflamed with a small amount of purulent discharge at os. Motion tenderness elicited. Uterus: normal size, no tenderness or masses. Adnexa: very tender on left.
Hct 39%, WBC 16.000. Culdocentesis yielded 20 ml of bloody, foul smelling, purulent fluid that was sent for aerobic and anaerobic culture. Gram stain of the fluid revealed gram-negative diplococci. You suspect PID caused by N. gonorrhoeae.

If you send a specimen for the culture, which of the following would most likely be the laboratory result?

1 Oxidase-positive colonies of gram-negative diplococci, maltose fermentation -
2 Oxidase-negative colonies of gram-negative diplococci, maltose fermentation -
3 Oxidase-positive colonies of gram-positive diplococci, maltose fermentation -
4 Oxidase-positive colonies of gram-positive diplococci, maltose fermentation +
5 Oxidase-positive colonies of gram-negative diplococci, maltose fermentation +

A

Oxidase-positive colonies of gram-negative diplococci, maltose fermentation -

Species
Gram stain
Oxidase
Polysaccharide capsule
Maltose fermentation
Neisseria meningitidis
Gram-negative diplococci
+
+
+
Neisseria gonorrhoeae
Gram-negative diplococci
+
_
_

115
Q

A 27-year-old woman is brought to the ER. She was well until today, when severe headache and vomiting suddenly began, accompanied by fever. T 39,6° C, BP 120/80, P 92. The patient appeared acutely ill. Physical exam revealed marked nuchal rigidity. Hct 40%, WBC 19,000; differential 16% bands, 80% polys, 4% monos. You suspect meningitis and do a spinal tap. Gram stain of the spinal fluid reveals many neutrophils and many gram-negative diplococci. Specimens of spinal fluid were sent for glucose, protein, and culture. Which one of the following is the most likely cause of the infection?

1 N. meningitidis
2 Haemophilus influenzae
3 Streptococcus pneumoniae
4 Corynebacterium diphtheriae
5 Bordetella pertussis

A

N. meningitidis

Neisseriae are gram-negative cocci that resemble paired kidney beans. Meningococcus has a prominent polysaccharide capsule that enhances virulence by its antiphagocytic action and induces protective antibodies. N. meningitidis mainly causes meningitis and meningococcemia. The most severe form of meningococcemia is the life-threatening Waterhouse-Friderichsen syndrome. It is characterized by high fever, shock, widespread purpura, disseminated intravascular coagulation, and adrenal insufficiency. A presumptive diagnosis of meningococcal meningitis can be made if gram-negative cocci are seen in a smear of spinal fluid. The organism grows best on chocolate agar incubated at 37° C in a 5% CO2 atmosphere. A presumptive diagnosis of Neisseria can be made if oxidase-positive colonies of gram-negative diplococci are found.
Haemophilus influenzae is a small gram-negative rod with a polysaccharide capsule. It is the leading cause of meningitis in young children, an important cause of upper respiratory tract infections and sepsis in children, and it causes pneumonia in adults, particularly those with chronic obstructive lung disease.
Streptococcus pneumoniae are gram-positive, encapsulated lancet-shaped coca arranged in pairs (diplococci) or short chains. Pneumococci cause pneumonia, bacteremia, meningitis, and infections of the upper respiratory tract, such as otitis and sinusitis. Mortality rate is high in elderly, immunocompromised (especially splenectomized), or debilitated patients. They should be immunized with the polyvalent polysaccharide vaccine.
Bordetella pertussis is a small, coccobacillary, encapsulated gram-negative rod. It causes whooping cough (pertussis).
Corynebacterium diphtheriae is gram-positive rod that appears club-shaped and is arranged in palisades or in V- or L-formations. C. diphtheriae causes diphtheria.

116
Q

Antibodies secreted by the host protect it in a variety of ways. There are a number of mechanisms with which bacteria and other microorganisms avoid inactivation by host antibodies, or even avoid eliciting antibodies. What is one example of a substance found in the cell wall of Staphylococcus aureus called?

1 Protein A
2 L-selectin
3 Pilin
4 Defensin
5 Streptolysin

A

Protein A

Protein A is present in the cell wall of S. aureus and other Staphylococcus species. Each molecule of Protein A binds strongly to two molecules of IgG via the Fc portion, and there are about 80,000 binding sites on each bacterium. This binding interferes with the opsonization and phagocytosis of staphylococci. Researchers have exploited this discovery and use Protein A to map the Fc portion of immunoglobulins and use Protein A as an immunological reagent.
Currently three members of the selectin family are known. These are E-selectin, P-selectin, and L-selectin. E-selectin is often induced on endothelial cells, P-selectin is found in platelets and endothelial cells, and L-selectin is leukocyte-specific. They are all carbohydrate-binding lectins that recognize specific sugar groups. L-selectin also binds to sulfated structures including sulfatide glycolipids. It is expressed on naïve T cells, where it binds to the carbohydrate moiety of mucin-like molecules called vascular addressins, which are expressed on the surface of vascular endothelial cells. The interaction between L-selectin and the vascular addressins is responsible for the specific homing of naïve T cells to lymphoid organs.
Pili are threads or fine hairs protruding from bacterial surfaces. They assist bacteria in establishing colonies and can be conferred on bacteria by plasmids. Pilin is the protein subunit of pili and consists of constant, variable, and hypervariable regions analogous with immunoglobulin molecules.
Defensins are highly abundant and variably cationic peptides that possess antimicrobial, cytotoxic, and chemoattractant properties and equip mammalian phagocytes for participation in host defense and inflammatory processes. They are small and have three intramolecular disulfide bonds. Defensins are active against a range of pathogens as diverse as S. aureus, P. aeruginosa, E. coli, C. neoformans, and Herpes simplex virus. Both neutrophils and macrophages contain defensin peptides.
Some bacteria, as they multiply in tissues, release soluble materials that are lethal for phagocytes. Pathogenic streptococci release hemolysins or streptolysins that lyse red blood cells and have an important toxic action on polymorphs and macrophages. Within one to two minutes of its addition to polymorphs, streptolysin O causes the polymorph granules to explode and their contents are discharged into the cell cytoplasm.

117
Q

A 22-year-old sexually active man develops pain when he urinates. He also notices a discharge when he urinates. He goes to his family doctor. A Gram stain of the urethral discharge is done and intracellular diplococci are visualized. What is the most likely diagnosis?

1 Primary syphilis
2 Condyloma acuminatum
3 Herpes genitalis
4 Chancroid
5 Gonorrhea

A

Gonorrhea

This man has signs and symptoms of gonorrhea. Gonorrhea can present with symptoms of urethritis. A purulent discharge and dysuria are consistent with gonorrhea. Intracellular diplococci on a Gram stain of the urethral discharge is also consistent with the diagnosis of gonorrhea.
In primary syphilis, there is a painless chancre along with painless regional lymphadenopathy. Darkfield examination of a lesion will show the spirochetes in the majority of cases. Gram stain would not show diplococci.
Condyloma acuminatum is the cause of genital warts. These are caused by human papilloma virus. Gram stain would not show diplococci.
The lesions caused by herpes genitalis are vesicular lesions on an erythematous foundation. Soft painful adenopathy can be seen with herpes genitalis as well as fever. Gram stain would not show diplococci.
The ulcer seen with chancroid would be painful. There would be painful tender nodes with chancroid. Gram stain would not show diplococci.

118
Q

Case
A 16-year-old girl presents with sore throat and headache. She started feeling poorly when she woke up this morning. Her immunizations are current. Temperature is 102°F. Physical examination is remarkable for erythema in the posterior pharynx and palatal petechiae. She also has tender anterior cervical lymphadenopathy.

What organism is the most likely cause of her condition?

1 Group A Streptococcus
2 Corynebacterium diphtheriae
3 Adenovirus
4 Rhinovirus
5 Candida albicans

A

Group A Streptococcus

Group A Streptococcus is the most common cause of bacterial pharyngitis. The patient has fever, palatal petechiae, and tender anterior cervical adenopathy, all of which are highly suggestive of bacterial pharyngitis.
Corynebacterium diphtheriae is an uncommon cause of sore throat in vaccinated children. Patients present with a gray pseudomembrane on examination of the throat.
Viral pharyngitis (adenovirus and rhinovirus) can present with erythema and tender anterior cervical adenopathy, but the palatal petechiae are consistent with GAS pharyngitis.
Candida albicans of the throat typically presents with thin exudate, but it is not common unless a patient is immunocompromised or has had recent antibiotic treatment.